SlideShare uma empresa Scribd logo
1 de 20
Baixar para ler offline
XAT Question Paper 2008

Instructions :

1. This question paper consists of three sections A, B and C with 38, 38 and 44 questions respectively, i.e. a total
   120 questions. Total time for the test is 120 minutes.

2. You are required to answer questions from all three sections and expected to maximize scores in each section.

3. Each correct answer will be awarded 1 (one) mark. NEGATIVE MARKS (one fourth of a mark) may be deducted
   for the first six incorrect answers in each section and 0.5 (half a mark) for each incorrect answer thereafter.



SECTION A: VERBAL AND LOGICAL ABILITY

Analyse the passage given and provide an                    buying more confident in buying, and much more willing
appropriate answer for the question nos. 1 through          to tolerate things like waiting in line. On the other hand,
6 that follow.                                              being in a mood makes buying behaviour in the “right
                                                            mood” by the use of music and friendly staff or, say,
Every conscious mental state has a qualitative character
                                                            opens bakeries in shopping malls that delight the passer-
that we refer to as mood. We are always in a mood that
                                                            by with the smell of fresh bread.
is pleasurable or unpleasurable to some degree. It may
                                                            Thayer views moods as a mixture of biological and
be that bad moods relate to their being too positive
                                                            psychological influences and, as such, a sort of clinical
reinforcement in a person’s current life and too many
                                                            thermometer, reflecting all the internal and external
punishments. In any case, moods are distinguished from
                                                            events that influence us. For Thayer, the key components
emotions proper by not being tied to any specific object.
                                                            of mood are energy and tension in different combinations.
But, this distinction is not watertight, in that emotions
                                                            A specific mixture of energy and tension, together with
need not be directed at objects that are completely
                                                            the thoughts they influence, produces moods. He discuss
specific (we can be angry just at people generally) while
                                                            four mood states:
there is always a sense of a mood having a general
objective like the state of the world at large. Moods       •   Calm - energy: he regards this as the optimal
manifest themselves in positive or negative feelings that       mood of feeling good
are tied to health, personality, or perceived quality of    •   Calm - tiredness: he regards this as feeling little
life. Moods can also relate to emotions proper as in the        tired without any stress, which can be pleasant.
aftermath of an emotional incident such as the failure to   •   Tense - energy: involves low level of anxiety
secure a loan. A mood on this basis is the mind’s               suited to a fight - or - flight disposition.
judgment on the recent past. For Goldie, emotion can        •   Tense - tiredness: is mixture of fatigue and
bubble up and down within a mood, while an emotion              anxiety, which underlines the unpleasant feeling
can involve characteristics that are non-object specific.       of depression.

What is important for marketing is that moods colour        People generally can “feel down” or “feel good” as result
outlook and bias judgements. Hence the importance of        of happenings in the world around them. This represents
consumer confidence surveys, as consumer confidence         the national mood. People feel elated when the national
typically reflects national mood. There is mood-            soccer team wins an international match or depressed
congruence when thoughts and actions fall in-line with      when their team has lost. An elated mood of calm -energy
mood. As Goleman says, there is a “constant stream of       is an optimistic mood, which is good for business.
feeling” that runs “in perfect to our steam of thought”.    Consumers, as socially involved individuals, are deeply
Mood congruence occurs because a positive mood              influenced by the prevailing social climate. Marketers
evokes pleasant associations that lighten subsequent        recognize the phenomenon and talk about the national
appraisals (thoughts) and actions, while a negative         mood being, say for or against conspicuous consumption.
arouses future judgment and behavior. When consumers        Moods do change, though. Writing early in the nineteenth
are in a good mood, they are more optimistic about          century.

XAT – 2008                                                                                                           1
Toqueville describers an American elite embarrassed by                 but not with action.
the ostentation of material display; in the “Gilded Age”,           E. When moods are synchronous with action but
sixty years later, many were only too eager to embrace                 not with though.
a materialistic vulgarity. The problem lies in anticipating   4.    Implication and Proposition are defined as follows:
changes in national mood, since a change in mood                    Implication: a statement which follows from the
affects everything from buying of equities to the buying            given text.
of houses and washing machines. Thayer would argue                  Proposition: a statement which forms a part of
that we should be interested in national events that are            the given text.
likely to produce a move towards a tense - tiredness                Consider the two statements below and decide
state or toward a calm - energy state, since these are              whether they are implications or propositions.
the polar extremes and are more likely to influence                 I. The marketers should understand and make
behavior. Artists sensitive to national moods express the               use of moods and emotions in designing and
long-term changes. An example is the long - tem                         selling products and services.
emotional journey from Charles Dickens’ depiction of                II. Consuming is nothing but way of feeling the
the death of little Nell to Oscar Wilde’s cruel flippancy               void in consciousness.
about it. “One would have to have a heart of stone not to           A. Both statements are implications.
laugh at the death of little Nell”. Which reflects the mood         B. First is implication, second is proposition.
change from high Victorian sentimentality to the acerbic            C. Both are propositions.
cynicism of the end of the century, as shown in writers             D. First is propositions, second is implication.
like Thomas Hardy and artists like Aubrey Beardsley.                E. Both are neither implication nor proposition.
Whenever the mind is not fully absorbed, consciousness
is no longer focused and ordered. Under such conditions       5.    Which statements from the ones given below are
the mind falls into dwelling on the unpleasant, with a              correct?
negative mood developing.                                           1. In general, emotions are object specify
                                                                    2. In general, moods are not object specific.
Csikszentmihalyi argues that humans need to keep                    3. Moods and emotions are same.
consciousness fully active is what influences a good deal           4. As per Thayer, moods are a mix of biological
of consumer behaviour. Sometimes it does not matter                    and psychological influences
what we are shopping for - the point is to shop for                 A. 1, 2, 3              B. 2, 3, 4
anything, regardless, as consuming is one way to respond            C. 1, 2, 4              D. 2, 4, 3
to the void in consciousness when there is nothing else             E. All four are right
to do.
                                                              6.    The statement “Moods provide energy for human
1.     Which one of the following statements best                   actions” is_____.
       summarizes the above passage?
                                                                    A. always right.       B. sometimes right.
       A. The passage highlights how moods affect
          nations.                                                  C. always wrong
       B. The passage draws distinction between moods               D. not derived from the passage.
          and emotions.                                             E. contradictory.
       C. Some writer influenced national moods               Directions (7 to 9): Carefully read the statements in
          through their writings.
                                                              the questions below and arrange them in a logical
       D. Thayer categorised moods into four sates.
       E. The passage highlights the importance of            order.
          moods and emotions in marketing.                    7:    1. So too it is impossible for there to be proposition
2.     Which of the following is the closest to                        of ethics. Proposition cannot express that is
       “conspicuous consumption” in the passage?                       higher.
       A. Audible consumption                                       2. The sense of the world must be outside the
       B. Consumption of material items for impressing                 world. In the world everything is as it is and
          others                                                       everything happens as it does happen: in it no
       C. Consumption driven by moods and emotions                     value exists-and if it did exist, it would have no
       D. Socially responsible consumption                             value. If there is any value that does have a
       E. Private but not public consumption                           value, it must lie outside whole sphere of what
3.     What is “moods congruence”?                                     happens and is the case. For all that happens
       A. When moods and emotions are synchronized.                    and is the case is accidental. What makes it
       B. When emotions are synchronous with actions                   non-accidental cannot lie within the world, since
          and thoughts.                                                if it did it would it self be accidental. It must lie
       C. When moods are synchronous with thoughts                     outside world.
          and actions.
       D. When moods are synchronous with thoughts

2                                                                                                             XAT – 2008
3. It is clear that ethics cannot be put into words.           A. ambivalence, prostate, immoral, amoral
         Ethics is transcendental.                                   B. ambivalence, prostrate, amoral, immoral
      4. All propositions are of equal value.                        C. ambiguity, prostrate, amoral immoral
      A. 4 - 2 - 1 - 3           B. 2 - 1 - 3 - 4                    D. ambivalence, prostrate, immoral, amoral
      C. 1 - 3 - 4 - 2           D. 4 - 3 - 1 - 2                    E. ambiguity, prostrate, immoral amoral
      E. 3 - 1 - 2 - 4
                                                              12.    It is not ________ democratic that the parliament
8:    1. The fact all contribute only to setting the                 should be _________ on issues and resort to
         problem, not to its solution.                               passing _______ rather than have an open debate
      2. How things are in the world is a matter of                  on the floor of the house.
         complete indifference for what is higher. God               A. quite, quite, ordinances
         does not reveal himself in the world.                       B. quite, quiet, ordnances
      3. To view the world sub specie aeterni is to view
                                                                     C. quiet, quite, ordnances
         it as a whole - a limited whole.
                                                                     D. quiet, quiet, ordinances
         Feelings the world as a limited whole - it is this
                                                                     E. quite, quiet, ordinances
         that is mystical.
      4. It is not how things are in the world that is        13.    In a case of acute ______, _______ membranes
         mystical, but that it exists.                               secrete excessive __________.
      A. 1 - 2 - 3 - 4             B. 2 - 1 - 3- 4                   A. sinus, mucous, mucous
      C. 3 - 1 - 4 - 2             D. 3 - 4 - 1 - 2                  B. sinus, mucus, mucous
      E. 2 - 1 - 4 - 3                                               C. sinusitis, mucous, mucus
                                                                     D. sinus, mucous, mucus
9:    1. The operation is what has to be done to one                 E. sinusitis, mucus, mucous
         proposition in order to make other out of it.
      2. Structure of proposition stands in internal          14.    If a person makes the statement: “ I never speak
         relations to one another.                                   the truth.” The person can be said to be
      3. In order to give prominence to these internal               ________.
         relations we can adopt the following mode of                A. speaking the truth.
         expression: we can represent a proposition as               B. lying.
         the result of an operation that produces it out             C. lying as well as speaking the truth
         of other propositions (which are bases of the               D. making a logically contradictory statement
         operation).                                                 E. partially speaking the truth and partially lying.
      4. An operation is the expression of a relation
                                                              Analyse the passage given and provide an
         between the structure of its result and of its
                                                              appropriate answer for the question nos. 15 through
         bases.
      A. 2 - 3 - 4 - 1         B. 1 - 2 - 3 - 4               18 that follow.
      C. 4 - 3 - 1 - 2         D. 2 - 1 - 3 - 4               Enunciated by Jung as an integral part of his psychology
      E. 4 - 1 - 2 - 3                                        in 1916 immediately after his unsettling confrontation
Directions (10 to 14): Choose the appropriate words           with the unconscious, the transcendent function was seen
to fill in the blanks.                                        by Jung as uniting the opposites, transforming psyche,
                                                              and central to the individuation process.
10.   Mark Twain was responsible for many striking,
      mostly cynical ____, such as “ Always do right.         It also undoubtedly reflects his personal experience in
      That will gratify some of the people, and astonish      coming to terms with the unconscious. Jung portrayed
      the rest”. ________ can sometimes end up as             the transcendent function as operating through symbol
      _____, but rarely would someone use them as             and fantasy and mediating between the opposites of
      an________.                                             consciousness and the unconscious to prompt the
      A. epitaphs, Epitaphs, epigrams, epigraph
                                                              emergence of a new, third posture that transcends the
      B. epigraphs, Epigraphs, epitaphs, epigraph
      C. epigrams, Epigrams, epigraphs, epitaph               two. In exploring the details of the transcendent function
      D. epigrams, Epitaphs, epigrams, epigraph               and its connection to other Jungian constructs, this work
      E. epitaphs, Epitaphs, epigraphs, epigram               has unearthed significant changes, ambiguities, and
                                                              inconsistencies in Jung’s writings. Further, it has identified
11.   A candidate in the medical viva voce exam faced
                                                              two separate images of the transcendent function: (1)
      a tinge of intellectual ______ when asked to spell
      the ___ gland. The fact that he carried notes on        the narrow transcendent function, the function or process
      his person would definitely be termed as ___ by         within Jung’s pantheon of psychic structures, generally
      factually, but may be termed as _______ by more         seen as the uniting of the opposites of consciousness
      generous sections of students.                          and the unconscious from which a new attitude emerges;
XAT – 2008                                                                                                                3
and (2) the expansive transcendent function, the root        Directions: (19 to 26): Go through the caselets below
metaphor for psyche or being psychological that              and answer the questions that follow.
subsumes Jung’ pantheon and that apprehends the most
fundamental psychic activity of interacting with the         Question No. (19 to 20): According to recent reports,
unknown or other. This book has also posited that the        CEOs of large organisations are paid more that CEOs
expansive transcendent function, as the root metaphor        of small organizations. It does not seem fair that just
for exchanges between conscious and the unconscious,         because a CEO is heading a big organization s/he should
is the wellspring from whence flows other key Jungian        be paid more. CEOs salary should be related to
structures such as the archetypes and the Self, and is       performance, especially growth in terms of sales and
the core of the individuation process. The expansive         profits. Of course, big organisations are more complex
transcendent function has been explored further by           than the small, but all CEOs require significant amount
                                                             of energy and time in managing organisations. There is
surveying other schools of psychology, with both death
                                                             no proof then CEOs of small organisations. All CEOs
and non-death orientations, and evaluating the
                                                             should be paid according to their performance.
transcendent function alongside structures or process
in those other schools which play similar mediatory and
                                                             19.   A person seeking to refute the argument might
/ or transitional roles.
                                                                   argue that
                                                                   A. CEOs should be paid equally.
15.   The above passage is most likely an excerpt from:            B. Managing big organisation is more challenging
      A. research note                                                that small .
      B. An entry on a psychopathology blog                        C. If CEOs of small companies perform well, the
      C. A popular magazine article                                   company would become big and so would be
      D. A newspapers article                                         CEOs salary.
      E. A scholarly treatise                                      D. CEOs who travel more should be paid more.
                                                                   E. Highly qualified CEOs should be paid more
16.   It can be definitely inferred from the passage                  because they have acquired difficult education.
      above that
      A. The expansive transcendent function would           20.   Which of the following, if true, would strengthen
         include elements of both the Consciousness                the speaker’s argument?
         and the Unconscious.                                      A. CEOs of small organisations come from good
      B. The transcendent is the core of the individuation            educational background.
         process.                                                  B. CEOs in big organisation take much longer to
      C. Archetypes emerge from the narrow                            reach top, as compared to their counterparts in
         transcendent function.                                       small organisations.
      D. The whole work, from which this excerpt is                C. CEOs of big organisations are very difficult to
         taken, primarily concerns itself with the                    hire.
         inconsistencies is Jung’s writings.                       D. A few big family businesses have CEOs from
      E. Jung’s pantheon of concept subsumes the root                 within the family.
         metaphor of psyche.                                       E. Big organisations contribute more towards
                                                                      moral development of society.
17.   A comparison similar to the distinction between
      the two images of the transcendent function would      Question no (21 to 22) Hindi ought to be the official
      be:                                                    language of India. There is no reason of the government
      A. raucous: hilarious B. synchronicity: ontology       to spend money printing documents in difference
      C. recession: withdrawal D. penurious: decrepit        languages, just to cater to people who cannot read/write
      E. None of these                                       Hindi. The government has better ways to spend tax
                                                             payers’ money. People across India should read/write
18.   As per the passage, the key Jungian structure -        Hindi or learn it at the earliest.
      other that the Self - that emerges from the
      expansive transcendent function may NOT be             21.   Which of the following, if true, would weaken the
      expressed as a(n):                                           speaker’s argument the most?
      A. Stereotype         B. Anomaly                             A. The government currently translates official
      C. Idealized model    D. Original pattern                       documents into more than eighteen languages.
      E. Epitome                                                   B. Hindi is the most difficult language in the world
                                                                      to speak.
4                                                                                                         XAT – 2008
C. People who are multilingual usually pay              24.   According to a recent research conducted by the
          maximum taxes.                                             district road planning department, ten percent
       D. Mot people who travel across India learn Hindi             students come with parents in cars, twenty percent
          within five years.                                         students use auto - rickshaws, twenty percent
       E. Making Hindi the official language is a                    students use taxis, forty percent students use the
          politically unpopular idea.                                school buses and ten percent students live in the
                                                                     hostel inside the school.
22.    United Nations members contribute funds,                      Which of the following is true about the above
       proportionate to their population, for facilitating           paragraph?
       smooth functioning of the UN. By 2010, India,                 A. It extends speaker’s argument using a analogy.
       being the most populous nation on the planet,                 B. It extends the speaker’s argument using
                                                                        statistical data.
       would contribute the maximum amount to the UN.
                                                                     C. It is similar to speaker’s argument.
       Therefore, official language of United Nations
                                                                     D. It contradicts the speaker’s argument using
       should be changed to Hindi.?
                                                                        statistical data.
       Which of the following is true?
                                                                     E. It concludes speaker’s argument by using
       A. The point above contradicts the speaker’s
                                                                        personalization.
          argument.
       B. The point above is similar to speaker’s              Question No (25 and 26): History, if viewed as a
          argument.                                            repository not merely of anecdotes or chronology, could
       C. The point above concludes speaker ’s                 produce a decisive transformation in the image of
          argument.                                            science by which we are now possessed. That image
       D. The point above extends the speaker ’s               has previously been drawn, even by scientists
          argument.                                            themselves, mainly from the study of finished scientific
       E. The point above strengthens the speaker’s            achievements as these are recorded in the classics and,
          argument.                                            more recently, in the textbooks from which each new
                                                               scientific generation learns to practice its trade.
Questions No (23 and 24): The Bistupur - Sakchi corner
needs a speed - breaker. Loyola school children cross          25.   Which of the following best summarizes the above
this intersection, on their way to the school, and many a            paragraph?
times do not check out for traffic. I get to read regular            A. Scientific achievements are record in classics
reports of cars and other vehicles hitting children. I know             and text books.
that speed - breakers are irritating for drivers and I know          B. Different ways of looking at history can
that children cannot be protected from every danger,                    produce altogether different knowledge.
but this is one of the worst intersections in town. There            C. History of science can be inferred from
needs to be a speed - breaker so that vehicles have to                  finished scientific achievement
slow down and the children be made safer.                            D. Text books may be biased.
                                                                     E. All of above.
23.    Which of the following arguments is used in the
                                                               26.   Which of the following statements is the author
       above passage?
                                                                     most likely to agree with?
       A. Analogy - comparing the intersection to                    A. History of science presents a scientific way of
          something dangerous                                           looking at scientific development and thus
       B. Statistical analysis - noting the number of                   contributes to progress in science.
          children hit by vehicles                                   B. History of science should contain only the
       C. Personalization - telling the story of one child’s            chronology of the scientific achievements.
          near accident at the intersection                          C. More number of scientific theories results in
       D. Attack - pointing out people who are against                  more number of publications, which benefits
          speed - breakers as being uncaring about                      publishers.
          children                                                   D. History of science should purposely present
       E. Emotive - referring to the safety of children to              different images of science to people.
          get people interested                                      E. History of science can present multiple
                                                                        interpretations to people regarding the process
                                                                        of scientific development.


XAT – 2008                                                                                                           5
Direction (27 and 28): Go through the caselets below         not a single entity but an amalgamation, a “construct”
and answer the questions that follow.                        arising from the conjoining of innumerable, discrete parts.
                                                             Modern scholarship has, quite properly, tended to explore
27.   Goodricke Group Ltd is planning to give top priority   these elements in isolation. (In part, this trend represents
      to core competence of production and marketing         the conscious reversal of the stance taken by an earlier
      of tea in 2007. The company intends to increase        generation of scholars whose work reified India into a
      the production of orthodox varieties of tea.           monolithic entity - a critical element in the much maligned
      Goodricke is planning to invest Rs. 10 crore to        “Orientalist” enterprise). Nonetheless, the representation
      modernise the factories. The company has               of India as a singular “Whole” is not an entirely capricious
      announced a net profit of Rs. 5.49 crore for 2006      enterprise; for India is an identifiable entity, united by - if
      as against Rs. 3.76 crore in 2005.                     not born out of - certain deep and pervasive structures.
      Which of the following can be deduced from the         Thus, for example, the Hindu tradition has long
      caselet?                                               maintained a body of mythology that weaves the
      A. Production and marketing is core competence         disparate temples, gods, even geographic landscapes
         of Goodricke Group.                                 that exist throughout the subcontinent into a unified, albeit
      B. Increase in production of existing products         syncretic, whole.
         enhances core competence.
      C. Core competence leads to modernization.             In the realm of thought there is no more pervasive,
      D. Goodricke has given top priority to production      unifying structure than karma. It is the “doctrine” or “law”
         because it has earned net profit of Rs. 5.49        that ties actions to results and creates a determinant
         crore.                                              link between an individual’s status in this life and his or
      E. Core competence can be used for furthering          her fate in future lives. Following what is considered to
         company’s interests.                                be its appearances in the Upanishads, the doctrine
                                                             reaches into nearly every corner of Hindu thought.
28.   The author reflects on the concept of Blue Ocean       Indeed, its dominance is such in the Hindu world view
      Strategy. He explains that this concept delivers       that karma encompasses, at the same time, life-affirming
      an instinctive framework for developing                and life-negating functions; for just as it defines the world
      uncontested market space and making the                in terms of the “positive” function of delineating a doctrine
      competition irrelevant. The author remarks that        of rewards and punishments, so too it defines the world
      the Blue Ocean Strategy is about having the best       through its “negative” representation of action as an all
      mix of attributes that result in creation of           but inescapable trap, an unremitting cycle of death and
      uncontested market space and high growth, and          rebirth.
      not about being the best.
      The above paragraph appears to be an attempt           Despite - or perhaps because of - karma’s ubiquity, the
      at                                                     doctrine is not easily defined. Wendy Doniger O’ Flaherty
      A. reviewing an article or a book on Blue Ocean        reports of a scholarly conference devoted to the study
         strategy.                                           of karma that although the participants admitted to a
      B. defining Blue Ocean strategy.                       general sense of the doctrine’s parameters, considerable
      C. developing the framework for Blue Ocean             item was in a “lively but ultimately vain attempt to define
         strategy.                                           ... karma and rebirth”. The base meaning of the term
      D. highlighting how Blue Ocean strategy leads          “Karma” (or, more precisely, in its Sanskrit stem form,
         to better returns.                                  Karman a neuter substantive) is “action”. As a doctrine,
      E. None of above.                                      Karma encompasses a number of quasi - independent
                                                             concepts: rebirth (punarjanam), consequence (phala,
Analyse the following passage and provide an                 literally “fruit,” a term that suggest the “ripening” of actions
appropriate answer for the question nos. 29 through          into consequences), and the valuation or “ethic-ization”
35 that follow.                                              of acts, qualifying them as either “good” (punya or
                                                             sukarman) or “bad” (papam or duskarman).
India is renowned for its diversity. Dissimilitude abounds
in every sphere from the physical elements of its land       In a general way, however, for at least the past two
and people to the intangible workings of its beliefs and     thousand years, the following (from the well known text,
practices. Indeed, given this variety, India appears to be   the Bhagvata Purana) has held true as representing the

6                                                                                                              XAT – 2008
principal elements of the Karma doctrine: “The same           34.   As per the author, which of the following
person enjoys the fruit of the same sinful or a meritorious         statements is wrong?
act in the next world in the same manner and to the                 A. India is a diverse country.
manner and extent according to the manner and extent                B. Doctrine of Karma runs across divergent Hindu
to which that (sinful or meritorious) act has been done                thoughts.
by him in this world.” Nevertheless, depending on the               C. Doctrine of Karma has a rich scholarly
doctrine’s context, which itself ranges from its                       discourse.
appearance in a vast number of literary sources to its              D. Modern scholars have studied Hinduism as a
usage on the popular level, not all these elements may                 syncretic whole.
be present (though in a general way they may be implicit).          E. Scholars could not resolve the meaning of
                                                                       Karma.
29.    The orientalist perspective, according to the
       author:
                                                              35.   Which of the following, if true, would be required
       A. Viewed India as country of diversity.
                                                                    for the concept of Karma - as defined in Bhagavata
       B. Viewed India both as single and diverse entity.
                                                                    Purana - to be made equally valued across
       C. Viewed India as if it was a single and unitary
                                                                    different space-time combinations?
          entity devoid of diversity.
                                                                    A. The information of the past actions and the
       D. Viewed India as land of ?Karma.
                                                                       righteousness of each action would be
       E. Viewed India in the entirely.
                                                                       embodied in the individual.
30.    “Reify” in the passage means:                                B. Karma is judged based on the observers’
       A. Reversal of stance                                           perception, and hence the observer is a
       B. Unitary whole                                                necessary condition for its validity.
       C. To make real out of abstract                              C. Karma is an orientalist concept limited to
       D. Diversity                                                    oriental countries.
       E. Unity in diversity                                        D. Each epoch will have its own understanding of
                                                                       Karma and therefore there can not be uniform
31.    “Ethic-ization” in the passage means                            validity of the concept of Karma.
       A. Process making something ethical                          E. Each space-time combination would have
       B. Converting unethical persons into ethical                    different norms of righteousness and their
       C. Teaching ethics                                              respective expert panels which will judge each
       D. Judging and evaluation                                       action as per those norms.
       E. None of these

32.    Consider the following statements:                     Analyse the passage given and provide an
       1. Meaning of Karma is contextual.                     appropriate answer for the question nos. 36 through
       2. Meaning of Karma is not unanimous.                  38 that follow.
       3. Meaning of Karma includes many other quasi
          independent concepts.                               Deborah Mayo is a philosopher of science who has
       4. Karma also means actions and their rewards.         attempted to capture the implications of the new
                                                              experimentalism in a philosophically rigorous way. Mayo
       Which of the above statements are true?                focuses on the detailed way in which claims are validated
       A. 1, 2, 3                B. 2, 3, 4                   by experiment, and is concerned with identifying just
       C. 1, 3, 4                D. None of these             what claims are borne out and how. A key idea underlying
       E. All the four are true.                              her treatment is that a claim can only be said to be
                                                              supported by experiment if the various ways in which
33.    The base meaning of “Karma, is:                        the claim could be at fault have been investigated and
       A. Reward and punishment.                              eliminated. A claim can only be said to be borne out by
       B. Only those actions which yield a “phala”.           experiment and a severe test of a claim, as usefully
       C. Any action.                                         construed by Mayo, must be such that the claim would
       D. Ripening of actions into consequences.              be unlikely to pass it if it were false.
       E. None of these.
                                                              Her idea can be explained by some simple examples.
                                                              Suppose Snell’s law of refraction of light is tested by

XAT – 2008                                                                                                           7
some very rough experiments in which very large                    B. Precise measurement is both a necessary and
margins of error are attributed to the measurements of                sufficient condition to ensure validity of
angels of incidence and refractions, and suppose that                 conclusion resulting from an experiment.
the results are shown to be compatible with the law within         C. Experimental data might support multiple
those margins of error. Has the law been supported by                 theoretical explanations at the same time,
experiments that have severely tasted it? From Mayo’s                 hence validity of theories needs to be tested
[perspective the answer is “no” because, owing to the                 further.
roughness of the measurements, the law of refraction               D. Precise      measurement        along    with
would be quite likely to pass this test even if it were               experimenter’s knowledge of the theory
false and some other law differing not too much from                  underpinning the experiment is sufficient to
Snell’s law true. An exercise I carried out in my school-             ensure the validity of conclusions drawn from
teaching days serves to drive this point home. My                     experiment.
students had conducted some not very careful                       E. All of these
experiments to test Snell’s law. I then presented them
with what had been suggested in antiquity and mediaeval      37.   As per Mayo’s perspective, which of the following
times, prior to the discovery of Snell’s law and invited           best defines the phrase “scientific explanation”?
the students to test them with the measurements they               A. One which is most detailed in its explanation
had used to test Snell’s law; because of the wide margins             of natural phenomena.
of error they had attributed to their measurements, all of         B. One which survives examinations better than
these alternative laws pass the test. This clearly brings             other explanations.
out the point that the experiments in question did not             C. One which has been thoroughly tested by
constitute a severe test of Snell’s law. The law would                scientific experts.
have passed the test even if it were false and one of the          D. One which refutes other explanations
historical alternatives true.                                         convincingly.
                                                                   E. All of these.
36.   Which of the following conclusion can be drawn
      from the passage?                                      38.   The author’s use of Snell’s law of refractions to
      A. Precise measurement is a sufficient condition             illustrate Mayo’s perspective can best said to be
         to ensure validity of conclusions resulting from          A. Contrived.           B. Premeditate.
         an experiment.                                            C. Superfluous.         D. Illustrative
                                                                   E. Inadequate.




SECTION B: ANALYTICAL REASONING & DECISION MAKING


Directions (Questions No. 39 to 59): Each group of           schedule MUST satisfy the following restrictions:
questions in this section is based on a set of
conditions. In answering some of the questions, it           She conducts quizzes on exactly three mornings.
may be useful to draw a rough diagram. Choose the            If she conducts quizzes on Monday, she does not conduct
response that most accurately and completely                 a quiz on Tuesday.
answers each question and blacken the
corresponding space on your answer sheet.                    She lectures in the afternoon on exactly two consecutive
                                                             calendar days.
Questions No 39 42: Professor Mukhopadhay works
only on Mondays, Tuesdays, Wednesdays, Fridays and           She evaluates quizzes on exactly one morning and three
Saturdays. She performs four different activities-           afternoons.
lecturing, conducting quizzes, evaluating quizzes and
working on consultancy projects. Each working day she
                                                             She works on consultancy project on exactly one
performs exactly one activity in the morning and exactly
                                                             morning.
one activity in the afternoon. During each week her work
                                                             On Saturday, she neither lectures nor conducts quizzes.
8                                                                                                        XAT – 2008
39.    On Wednesdays, the Professor could be                  medical institutes, two management institutes, and two
       scheduled to                                           technical institutes. These eight institutions are located
       A. Conduct a quiz in the morning and lecture in        as follows:
          the afternoon.
       B. Work on a consultancy project in the morning        No institution is in more than one of the states.
          and conduct a quiz in the afternoon.                None of the states contains more than one management
       C. Lecture in the morning and evaluate quizzes         institute, and none contains more than one technical
          in the afternoon.                                   institute.
       D. Conduct a quiz in the morning and work on           None of the states contains both a management institute
          consultancy project in the afternoon.               and a technical institute.
       E. Evaluate quizzes in the morning and evaluate        Each management institute is located in a state that
          quizzes in the afternoons.                          contains at least one medical institute.
                                                              The technical institutes are located in two states that do
40.    Which one of the following statements must be          not share a common boundary.
       true?                                                  State 3 contains a technical institute, and State 6 contains
       A. There is one day on which she evaluates             a management institute.
          quizzes both in the morning and in the
          afternoon.                                          43.    Which one of the following could be true?
       B. She works on consultancy project on one of                 A. State 1 contains exactly one technical institute
          the days on which lecturers.                               B. State 2 contains exactly one management
       C. She lectures on one of the days on which she                  institute
          conducts quiz.                                             C. State 5 contains exactly one technical institute
       D. She works on consultancy project on one of                 D. State 6 contains exactly one technical institute
          the days on which she evaluates quizzes.                   E. State 1 contains exactly one medical institute
       E. She lecturers on one of the days on which
          evaluates quizzes.                                  44.    A complete and accurate list of the states, any
                                                                     one of which could contain the management
41.    If the Professor conducts a quiz on Tuesday, then             institute that is not in State 6, would be ________.
       her schedule for evaluating quizzes could be                  A. 1, 4                     B. 2, 4
       A. Monday morning, Monday afternoon, Friday                   C. 1, 4, 5                  D. 4, 5
           morning, Friday afternoon.                                E. 1, 2, 4, 5
       B. Monday morning, Friday afternoon, Saturday
           morning, Saturday afternoon                        45.    If each of the six states contains at least one of
       C. Monday afternoon, Wednesday morning,                       the eight institutions, then which one of the
           Wednesday afternoon, Saturday afternoon                   following must be true?
       D. Wednesday morning, Wednesday afternoon,                    A. There is management institute in state 1
           Friday afternoon, Saturday afternoon                      B. There is a medical institute in state 2
       E. Wednesday afternoon, Friday afternoon,                     C. There is a medical institute in state 3
           Saturday morning, Saturday afternoon                      D. There is a medical institute in state 4
                                                                     E. There is a management institute in state 4
42.    Which one of the following must be a day on which
       Professor lectures?                                    46.    If one of the states contains exactly two medical
       A. Monday               B. Wednesday                          institutes and exactly one technical institute, then
       C. Friday               D. Tuesday                            which combinational of three states might contain
       E. Saturday                                                   no medical institute?
                                                                     A. 1, 4, 5               B. 2, 3, 5
Question No 43 to 46: Six square states having equal                 C. 2, 4, 6               D. 1, 3, 5
area in a country are located in North-South directions              E. 4, 5, 6
in two columns next to each other. States are located in
the given order, State 1, State 3, and State 5 are on the
western side and State 2, State 4, and State 6 are on the
eastern side. Within the six states, there are exactly four

XAT – 2008                                                                                                              9
Question No 47 to 50: During a four-week period each           of exactly six words and satisfies the following conditions:
one of seven previously unadvertised products - G, H J,        The six words are written from left to right on a single
K, L, M and O - will be advertised,. A different pair of       line in alphabetical order.
these products will be advertised each week. Exactly           The sentence can start with any word, and successive
one of the products will be a members of two of these          word is formed by applying exactly one of three
four pairs. None of the other products gets repeated in        operations to the preceding world: delete one letter; add
any pair.                                                      a letter; replace a one letter with another.
Further , the following constraints must be observed:          At the most three of the six words can begin with the
J is not advertised during a given week unless H is            same letter.
advertised during the immediately preceding week.              Except for the first word each word is formed by a
The product that is advertised twice is advertised during      different operation used for the preceding word.
week 4 but is not advertised during week 3.
G is not advertised during a given week unless either J        51.    Which one of the following could be a sentence
or O is also advertised that week.                                    in the word game?
K is advertised during one of the first two weeks.                    A. Bzaeak blaeak laeak paeak paea paean
O is one of the products advertised during week 3.                    B. Doteam goleam golean olean omean omman
                                                                      C. Crobek croeek roeek soeek sxoeek xoeek
47.    Which one of the following could be the schedule
                                                                      D. Feted freted reted seted seteg aseteg
       of the advertisements?
                                                                      E. Forod forol forols forpls orpls morpls
       A. Week 1: G, J; week 2 : K, L; week 3: O,
          M; week 4: H, L
                                                               52.    The last letter of the English alphabet that the
       B. Week 1: H, K; week 2: J, G; week 3: O, L;
                                                                      first word of a sentence in the word game can
          week 4: M, K
                                                                      begin with is
       C. Week 1: H, K; week 2: J, M; week 3: O, L;
                                                                      A. t                    B. w
          week 4: G, M
                                                                      C. x                    D. y
       D. Week 1: H, L; week 2: J, M; week 3: O, G;
                                                                      E. z
          week 4: K, L
       E. Week 1: K, M; week 2: H, J; week 3: O, G;
                                                               53.    If the first word in a sentence is “illicit” and the
          week 4: L, M
                                                                      fourth word is “licit”, then the third word can be
48.    If L is the product that is advertised during two of           A. Implicit                B. Explicit
       the weeks, which one of the following is a product             C. Enlist                  D. Inlist
       that MUST be advertised during one of the weeks                E. Elicit
       in which L is advertised?
       A. G                       B. H                         54.    If “clean” is the first word in a sentence and “learn”
       C. M                       D. K                                is another word in the sentence, then which one
       E. J                                                           of the following is a complete and accurate list of
                                                                      the positions “learn” could occupy?
49.    Which one of the following is a product that could
                                                                      A. Third
       be advertised in any of the four weeks?
                                                                      B. Second, third, fourth
       A. H                    B. L
                                                                      C. Third, fourth, fifth, sixth
       C. K                    D. L
                                                                      D. Third, fourth
       E. O
                                                                      E. Third, fourth, fifth
50.    Which one of the following is a pair of products
                                                               55.    If the first word in a sentence consists of five
       that could be advertisement during the same
                                                                      letters, then the maximum number of letters that
       week?
                                                                      the fifth word in the sentence could contain is
       A. M and O             B. G and H
                                                                      A. Four                  B. Five
       C. H and J             D. H and O
                                                                      C. Six                   D. Seven
       E. K and O
                                                                      E. Eight
Questions No 51 to 55: In a game, “words” (meaningful
                                                               Question No 56 to 59: There are exactly ten stores and
or meaningless) consist of any combination of at least
                                                               no other building on a straight street in Bistupur Market.
five letters of the English alphabets. A “sentence” consists
                                                               On the northern side of the street, from West to East,
10                                                                                                            XAT – 2008
are stores 1, 3, 5, 7 and 9; on the southern side of the        Go through the situation and the accompanying
street, also from West to east, are stores 2, 4, 6, 8 and       table, and pick up the best alternatively to answer
10. The stores on the northern side are located directly        Question nos. 60 to 62
across the street from those on the southern side, facing       Question No 60-61: There are five sets of digits - Set
each other in pairs, as follows: 1 and 2; 3 and 4; 5 and 6;     A, Set B, Set C, Set D, and Set E as shown in given
7 and 8; 9 and 10. Each store is decorated with lights in       diagram. Set A contains one digit, Set B contains two
                                                                digits, Set C contains three digits, set D contains two
exactly one of the following colours: green, red, and
                                                                digits, and Set E contains one digit.
yellow. The stores have been decorated with lights
                                                                Rearrange the digits, across the sets, such that the
according to the following conditions:                          number formed out of digits of Set C is multiple of the
No store is decorated with lights of the same colour as         numbers formed from digits in sets on either side. For
those of any store adjacent to it.                              example; in the given diagram, SET C is a multiple of
No store is decorated with lights of the same colour as         digits in SET A and SET B but not of SET D and SET E
those of the store directly cross the street from it.
Yellow lights decorate exactly one store on each side of                SET A SET B SET C SET D SET E
the street.                                                             7     28    196   34    5
Red lights decorate store 4.
Yellow lights decorate store 5.
                                                                60.   What is the minimum number of rearrangements
56.    Which one of the following could be an accurate                required to arrive at the solution? A rearrangement
       list of the colours of the lights that decorate stores         is defined as an exchange of positions between
       2, 4, 6, 8 and 10, respectively?                               digits across two sets. For example: when 1 from
       A. green, red, green, red, green                               set C is exchanged with 5 of set E, it is counted
       B. green, red, green, yellow, red                              as one rearrangement.
       C. green, red, yellow, red, green                              A. 2                       B. 5
       D. yellow, green, red, green, red                              C. 8                       D. 3
       E. yellow, red, green, red, yellow                             E. 7
57.    If green lights decorate store 7, then each of the
                                                                61.   Which of the following pair of digits would occupy
       following statements could be false
                                                                      set A and E?
       EXCEPT:
                                                                      A. 2 and 4              B. 2 and 6
       A. Green lights decorate store 2
       B. Green lights decorate store 10                              C. 3 and 6              D. 3 and 9
       C. Red lights decorate store 8                                 E. 4 and 8
       D. Yellow lights decorate store 2                        Question No. 62: Magic Box
       E. Red lights decorate store 9
                                                                                   5    15   1    16
58.    Which one of the following statements MUST be                               10   4    8    9
       true?                                                                       11   6    12   2
       A. Red lights decorate store 1                                              4    3    13   7
       B. Green lights decorate store 10                        Cut the square given above into four pieces along the
       C. Red lights decorate store 8                           lines and rearrange in such a manner that sum of all
       D. Yellow lights decorate store 8                        rows, columns and diagonals is equal to 34. One of the
       E. Yellow lights decorate store 10                       pieces, comprising 1 and 8, is shown in the diagram
                                                                given below
59.    Suppose that yellow lights decorate exactly two
       stores on the south side of the street and exactly                      1
       one store on the north side. If all other conditions                    8
       remain the same, then which one of the following
       statements Must be true?
       A. Green lights decorate store1                                How many numbers would be there in the largest
       B. Red lights decorate store 7                                 piece?
       C. Red lights decorate store 10                                A. 5               B. 6
       D. Yellow lights decorate store 2
                                                                      C. 9               D. 10
       E. Yellow light decorate store 8
                                                                      E. 8

XAT – 2008                                                                                                            11
Read the following situation and choose the best                    B. Reduce the number of female employees, as
possible alternative.                                                  it is a business requirement. She should not let
                                                                       anything affect her business.
Question No. 63: The surnames of four professionals                 C. Let the status quo continue.
are: Bannerji, Chatterji, Mukherji and Pestonji. Their              D. Henceforth hire only male employees.
professions are accountant, lawyer, dentist and doctor              E. She should close the business.
(not necessarily in this order). The accountant and lawyer
work in their offices, while the dentist and doctor work in   65.   You, a recruitment manager, are interviewing
their nursing homes. The accountant looks after                     Mayank, a hard- working young man, who has
Mukherji’s and Chatterji’s account. Chatterji, dose not             problems in speaking fluent English. He has
know Bannerji, although his nursing home is in the same             studied in vernacular medium schools and
street as Bannerji’s office. Chatterji is not a doctor.             colleges. Amongst the following options, what
What are the occupations of the four people.                        would you choose to do, if your company has
                                                                    vacancies?
       A. Bannerji- Doctor, Chatterji- Dentist,Mukherji-            A. I would hire him at all costs.
          accountant and Pestonji- Lawyer.                          B. I would hire him for the job he is good at, and
       B. Bannerji- Lawyer, Chatterji- Dentist, Mukherji-              provide training in other areas.
          accountant and Pestonji- Doctor.                          C. I would hire him for production or finance job
       C. Bannerji- Lawyer, Chatterji- Dentist, Mukherji-              but not for marketing job, which requires good
          Doctor and Pestonji- Accountant.                             communication skills.
       D. Bannerji- Doctor, Chatterji- Accountant,                  D. I would ask him to improve his communication
          Mukherji- Dentist and Pestonji- Lawyer.                      skills and come back again.
       E. Bannerji- Dentist, Chatterji- Lawyer, Mukherji-           E. I would not hire him as he might be a burden
          Doctor and Pestonji- Accountant.                             on organisation because of his poor
                                                                       communication skills.
Questions 64 - 67: Read the following situations and
choose the best possible alternative.                         66.   The city of Nagar has a population of 10 million,
                                                                    2 millions amongst whom are rich, 3 million poor
64.    Seema was a finance manager in an MNC and                    and 5 million belong to the middle class.
       felt that gender discrimination at the workplace             Saundarya Cosmetics manufactured and sold
       hampered her career growth. Frustrated, she quit             beauty product to the rich class at a premium
       the job and started a company. While starting her            price. Its products were very popular with
       company, Seema decided that she would have                   customers. Many people from the middle and poor
       equal proportion of males and females. Over the              segments of the population aspired to buy these
       next six years, Seema emerged as a very                      products but could not afford because of the high
       successful entrepreneur and expanded her                     prices. Of late, sales growth has been stagnating
       business to eight locations in the country.                  in the rich segment. Which of the following is the
       However, Seema recently started facing an ethical            best option for Saundarya Cosmetics to maximize
       dilemma because she realized that female                     long-term profits?
       employees were not willing to travel across cities           A. Sell the same products at lower prices to middle
       and work late hours, as the work required them to               and poor classes.
       do. Male employees did not hesitate undertaking              B. Sell similar products, of different quality
       such work. Seema started to feel the pressure of                standards with different brand names, to middle
       reducing the proportion of female employees. On                 classes and poor classes.
       the other hand, she is aware that equal                      C. Sell its products under different brand names
       representation was one of the strongest reasons                 to middle and poor classes.
       for her to have founded the company. What should             D. Continue to target rich only and hope that
       she do as a conscientious female entrepreneur?                  today’s middle class would be tomorrow’s rich
                                                                       class.
       A. See if unwilling female employees could be                E. Target middle class as it is the largest segment
          given assignments which do not require travel                and forget about the rich.
          and involve less overtime.

12                                                                                                        XAT – 2008
67.   A database software manufacturing company                    A. Apologize and fix up the bug for all customers
      found out that a product it has launched recently               even if it has to incur losses.
      had a few bugs. The product has already been                 B. Do not tell customers about bugs and remove
      bought by more than a million customers. The                    only when customers face problems, even if it
                                                                      means losses for the customers.
      company realized that bugs could cost its
                                                                   C. Keep silent and do nothing.
      customers significantly. However, if it informs the          D. Keep silent but introduce and improved product
      customers about the bug, it feared losing                       that is bug-free at the earliest.
      credibility. What would be the most ethical option           E. Take the product off the mark and apologize to
      for the company?                                                customers.

Go through the table that follows and pick up the best alternative to answer question no. 68 to 70.

             Name of the     Average of batmen Number of Number of times Number of times
              batsman          based on past     times      dismissed    scores more than
                                performance    dismissed around average      a century
                                                below 20
                 RD                  40            20           70               3
                 ST                  44            20           60              10
                 SG                  41            25           50              10
                 VS                  31            50           20              15
                 RU                  28            55           25              12
                 YS                  35            40           40              10
                 VV                  35            35           50               5
                 MK                  30            30           45               5
                 MT                  36            45           30              10
                 MD                  45            30           50              10

Teams A, B, C and D are participating in a cricket tournament. Team A has to pick up five batsmen out of the ten
available. All batsmen have played 100 matches each in the past. Past data indicates that C beats A 8 out of 10
times. B beats A 5 out of 10 times and D beats A 1 out of 10 times. The conditions for series are likely to be normal
and bowling strength of all teams is same. Manager of Team A, based on his past experience, feels that team should
take high risk against stronger opponents and low risk against weaker opponents for maximizing chances of wining
the game.

The average scores of the top 5 batmen for each team playing in the tournament are: C (270); B (215); D
(180) and A (215).

68.   Team A would play the third match with B. Based on the statistics above, whom should the manager choose
      so that A has maximum chances of winning?
      A. RD, RU, MU,VS,YS                          B. RD, VS, MT, RU, YS
      C. RD, VV,SG,VS, MD                          D. ST, RD,MK, MD, SG                E. SG, RU, YS, MK,VV

69.   Team A is playing its first match with team C. Based on the statistics above, whom should the manger choose
      so that the team has maximum chances of winning?
      A. RD, ST, SG, MD,YS                            B. VS, YS, RU,MD, MT
      C. RD, ST, SG, VS,MD                            D. YS, RU, VS, MK, MD                E. ST, VS, RU, MD, SG

70.   Team A would play the second match with D. Based on the statistics above, whom should the manger choose
      so that A has maximum chances of winning?
      A. RD, ST, MD,VS,YS                         B. ST, RD, VV, SG, MD
      C. RD, ST, SG, VS,MD                        D. SG, RU, YS, MK, MD                E. ST, RD, MK,MD,SG



XAT – 2008                                                                                                        13
Read the following caselet and choose the best                 71.   The most important causal factor for this entire
alternative (Question 71 - 76):                                      episode could be:
                                                                     A. Trying to follow a divide-and-rule policy in his
Mr. Rajiv Singhal, Chairman of the Board of Directors of
                                                                        dealings with the supervisors.
Loha India Ltd., (a steel manufacturing company) had
                                                                     B. Inconsistent dealings of Mr. Thakur with
just been visited by several other directors of the
                                                                        supervisors.
company. The directors were upset with recent actions
                                                                     C. Paternalistic approach towards mature
of the company president, Mr. Ganesh Thakur. They
                                                                        individuals in the organisation.
demanded that the board consider firing the president.
                                                                     D. Legalistic approach to employee problems.
                                                                     E. Inadequate standards for measurement of
Mr. Thakur, recently appointed as president, had
                                                                        supervisors’ on-job performance.
undertaken to solve some of the management-
employees problems by dealing directly with the
                                                               72.   The situation with Mr.Lal could have been avoided
individuals, as often as possible. The company did not
                                                                     if Mr. Thakur had
have a history of strikes or any other form of collective
                                                                     1. Delegated the task of negotiation of wage
action and was considered to have good work culture.
                                                                        contracts for night shift employees to Personnel
However, Mr. Thakur felt that by dealing directly with
                                                                        department.
individuals, he could portray the management’s concern
                                                                     2. Created a process for supervisors working the
for the employees. An important initiative of Mr. Thakur
                                                                        night shift so that they could have and
was to negotiate wages of the supervisors with each
                                                                        opportunity to interact with him.
supervisor. In these negotiation meetings, he would not
                                                                     3. Created an open door policy that would have
involve anyone else, including the Personnel Department
                                                                        allowed employees to see him without any
which reported to him, so as to take an unbiased decision.
                                                                        appointment.
After negotiation, a wage contract would be drawn up
                                                                     4. Postponed the decision of wage revision for
for each supervisor. This, he felt, would recognize and
                                                                        supervisors in the night shift for two months,
reward the better performers. Mr. Thakur successfully
                                                                        since supervisors were rotated on different
implemented the process for most of the supervisors,
                                                                        shifts after every two months.
except those working in night shift with that of supervisors
                                                                     The option that best arranges the above
of the day shift.
                                                                     managerial interventions in decreasing order of
                                                                     organisational impact is:
For several days Ram Lal, a night shift supervisor, had
                                                                     A. 4, 2, 3, 1             B. 4, 3, 2, 1
been trying to seek and appointment with
                                                                     C. 4, 3, 1, 2             D. 4, 1, 2, 3
Mr. Thakur about his wages. He was disgruntled, not
                                                                     E. 2, 3, 1, 4
only over his failure to see the president, but also over
the lack of discussions about his wage contract prior to       73.   The most likely premise behind Mr. Thakur’s
its being effected. As a family man with six dependents,             initiative regarding individualised meetings with
he felt his weekly wage should be higher than that granted           the supervisors seems to be
to him.                                                              A. Employee related policies should allow scope
                                                                        for bargaining by employees.
Last Thursday afternoon Ram Lal stopped by the                       B. Involvement of company’s president in wage
president’s office and tried to see him. Mr.Thakur’s                    problems of employees will lead to a better
secretary refused his request on the grounds that Mr.                   goodwill towards the management among the
Thakur was busy. Infuriated, Ram Lal stormed into the                   workers.
president’s office and confronted the startled Mr. Thakur,           C. Individual agreements with supervisors would
with stood up and told Ram Lal to get out of his office                 allow the management to prevent any possible
and express his grievance through official channel. Ram                 collective action by the supervisors.
Lal took a swing at the president, who in turn punched               D. Management will be able to force supervisors
Ram Lal on the jaw and knocked him unconscious.                         to accept lesser wages individually in this way.
                                                                     E. He would be able to know who the trouble
                                                                        makers in the plant are by interacting with the
                                                                        supervisors.

14                                                                                                         XAT – 2008
74.                     Out of the following, which one seems to be the             2. Finance department - for not taken into
                        most likely cause of Ram Lal’s grievance?                      confidence regarding the financial
                        A. His disappointment with the management’s                    consequences of the wage contracts.
                           philosophy of having one to one interaction as           3. Marketing department - for not being consulted
                           the supervisors were in a way being forced to               on the likely impact of the wage contracts on
                           accept the wage contracts.                                  the image of the company.
                        B. His being in the night shift had worked to his           4. Quality control - for not being able to give inputs
                           disadvantage as he could not interact with the              to Mr. Thakur on how to improve quality of steel
                           management regarding his problem.                           making process.
                        C. He was not allowed to meet chairman of the               5. Personnel department - for it was their work to
                           board of directors of the company.                          oversee wage policies for employees and they
                        D. Employment in the night shift forced him to stay            had been ignored by Mr. Thakur.
                           away from his family during the day time and             A. 1 + 2 + 3               B. 1 + 4 + 5
                           therefore he could not interact with his family          C. 1 + 2 + 5               D. 1 + 3 + 4
                           members much.                                            E. 3 + 4 + 5
                        E. All of these.
                                                                              76.   Which of the following managerial attributes does
75.                     Apart from the supervisors working the night shift,
                                                                                    Mr. Thakur seem to lack the most?
                        executives of which department will have most
                                                                                    A. Emotional instability under pressure.
                        justified reasons to be disgruntled with Mr.
                                                                                    B. Proactive problem solving.
                        Thakur’s initiative?
                                                                                    C. Ethical behaviour.
                        1. Production department - for not being consulted
                                                                                    D. Emotional stability under pressure.
                           regarding the behaviour of the supervisors on
                                                                                    E. Independent decision making.
                           the shop floor.


SECTION C: DATA INTERPRETATION AND QUANTITATIVE ABILITY

Note: All units of measurement are in centimetres,                                  A. Returns of stock X are directly proportional to
unless otherwise specified.                                                            mutual fund Y.
77.   Four digits of the number 29138576 are omitted                                B. Average returns from stock X and mutual fund
      so that the result is as large as possible.                                      Y are the same.
      The largest omitted digit is                                                  C. Stock X is less volatile than mutual fund Y.
      A. 9                   B. 8                                                   D. Stock X is more volatile than mutual fund Y.
      C. 7                   D. 6                                                   E. Stock X is inversely proportional to mutual fund
      E. 5                                                                             Y.
78.                     Interpret relationship between the returns of Stock
                                                                              Question 79 - 80: In second year, students at a business
                        X and Mutual Fund Y based on the following graph,
                                                                              school can opt for Systems, Operations, or HR electives
                        where percentage return of Stock X and Mutual
                                                                              only. The number of girls opting for Operations and the
                        Fund Y are given for sixteen days of a month.
                                                                              number of boys opting for Systems electives is 37.
                      1.2                                                     Twenty-two students opt for operations electives. Twenty
                       1                                                      girls opt for Systems and Operations electives.
  Percentage Return




                                                                              The number of students opting for Systems electives
                      0.8
                                                                              and the number of boys opting for Operations electives
                      0.6                                                     is 37. Twenty-five students opt for HR electives.
                      0.4
                                                                              79.   The number of students in the second year is ___?
                      0.2
                                                                                    A. 73                  B. 74
                       0                                                            C. 76                  D. 53
                            1 2 3 4 5 6 7 8 9 10 11 12 13 14 15 16                  E. 54
                                          Day of the Month
                                        Stock X   Mutual Fund Y
XAT – 2008                                                                                                                             15
80.   If 20% of the girls opt for HR electives, then the           A. Using the given statement, only conclusion I
      total number of boys in the second year is ___?                 can be derived.
      A. 50                    B. 52                               B. Using the given statement, only conclusion
      C. 51                    D. 53                                  II can be derived.
                                                                   C. Using the given statement, only conclusion
      E. 54
                                                                      III can be derived.
81.   ABCD is a rectangle with AD = 10. P is a point on            D. Using the given statement, all conclusions
      BC such that ∠ APD = 90°. If DP = 8 then the                    I, II and III can be derived.
      length of BP is ___?                                         E. Using the given statement, none of the three
                                                                      conclusions I, II and III can be derived.
      A. 6.4                 B. 5.2
      C. 4.8                 D. 3.6                                                                          b
                                                             86.   An operation “#” is defined by a # b = 1–
      E. None of the above                                         Conclusion I. (2 # 1) # (4 # 3) = –1      a

82.   Rajiv is a student in a business school. After every         Conclusion II. (3 # 1) # (4 # 2) = –2
                                                                   Conclusion III. (2 # 3) # (1 # 3) = 0
      test he calculates his cumulative average. QT and
      OB were his last two tests.83 marks in QT
                                                             87.   A, B, C and D are whole numbers such that
      increased his average by 2. 75 marks in OB further           A + B +C = 118         B + C + D = 156
      increased his average by 1. Reasoning is the next            C + D + A = 166        D + A + B = 178
      test, if he gets 51 in Reasoning, his average will           Conclusion I.
      be ___?                                                      A is the smallest number and A = 21.
      A. 59                     B. 60                              Conclusion II.
      C. 61                     D. 62                              D is the smallest number and D = 88.
                                                                   Conclusion III.
      E. 63
                                                                   B is the smallest number and B = 56.
83.   ABCD is a quadrilateral. The diagonals of ABCD
                                                             88.   Let X = {a, b, c} and Y = {1, m}.
      intersect at the point P. The area of the triangles          Consider the following four subsets of X × Y.
      APD and BPC are 27 and 12, respectively. If the              F1 = {(a, 1), (a, m), (b, 1), (c, m)}
      areas of the triangles APB and CPD are equal                 F2 = {(a, 1), (b, 1), (c, 1)}
      then the area of triangle APB is                             F3 = {(a, 1), (b, m), (c, m)}
      A. 12                     B. 15                              F4 = {(a, 1), (b, m)}
      C. 16                     D. 21                              Which one, amongst the choices given below, is
                                                                   a representation of functions from X to Y?
      E. 18
                                                                   A. F2 and F3               B. F1, F2 and F3
84.   If F (x, n) be the number of ways of distributing            C. F2, F3 and F4           D. F3 and F4
      “x” toys to “n” children so that each child receives         E. None of the above
      at the most 2 toys, then F(4, 3) = ___?                89.   In the figure, number in any cell is obtained by
      A. 2                      B. 6                               adding two numbers in the cells directly below it.
      C. 3                      D. 4                               For example, 9 in the second row is obtained by
      E. 5                                                         adding the two number 4 and 5 directly below it.
                                                                   The value of X - Y is
85.   In a cricket match, Team A scored 232 runs without
      losing a wicket. The score consisted of byes, wides                               68
      and runs scored by two opening batsmen: Ram
      and Shyam. The runs scored by the two batsmen                                           Y + 29
      are 26 times wides. There are 8 more byes than
      wides. If the ratio of the runs scored by Ram and
      Shyam is 6 : 7, then the runs scored by Ram is
      A. 88                     B. 96                                               9
      C. 102                    D. 112
      E. None of the above.                                        Y         4          5           2      X

For question 86 and 87, a statement is followed by
                                                                   A. 2                      B. 4
three conclusions. Select the answer from the
                                                                   C. 3                      D. 5
following options.                                                 E. 6
16                                                                                                      XAT – 2008
90.   For each p > 1, a sequence {An} is defined by            97.   If [x] denotes the greatest integer < = x, then
      A0 = 1 and An = pn + (–1)n An – 1 for each n ≥1. For            1  1 1   1 2                    1 98 
      how many integer values of p, 1000 is a term of                 3  +  3 + 99  +  3 + 99  + ...  3 + 99  =
                                                                                                             
      the sequence?
                                                                     A. 98                       B. 33
      A. 8                   B. 7                                    C. 67                       D. 66
      C. 5                   D. 4                                    E. 34
      E. None of the above.
                                                               98.   ABCD is a square. P is the midpoint of AB.
91.   If o < p< 1 then roots of the equation                         The line passing through A and Perpendicular to
      (1 – p) x2 + 4x + p = 0 are ___?                               DP intersects the diagonal at Q and BC at R.
      A. Both 0                                                      If AB = 2 then PR = ___?
      B. Real and both negative                                          1
                                                                     A.                     B. 2
      C. Imaginary                                                       2
      D. Real and both positive                                                                   3
                                                                     C. 1                   D.
      E. Real and of opposite sign.                                                              2
                                                                     E. None of the above.
92.   If x > 0, the minimum value of
                                                               99.   Two circles of radius 1 cm. touch at point P. A
              6                                                      third circle is drawn through the points A, B, and
          1     6 1  
      x + x  – x + 6 –2                                          C such that PA is the diameter of the first circle,
                  x                                              and BC - perpendicular to AP - is the diameter of
                            is ___?
                 3
            1     3 1                                            the second circle. The radius of the third circle is
        x + x  + x + 3                                           ___ cms.
                    x 
                                                                          9                            7
      A. 1                  B. 2                                     A.                           B.
      C. 3                  D. 6                                          5                            4
      E. None of the above.                                              5                                 10
                                                                     C.                           D.
93.   The number of possible real solution(s) of y in                    3                                 2
      equation y2 – 2y cos x + 1 = 0 is____?                         E. 2
      A. 0                    B. 1                             100. Consider a sequence –6, –12, 18, 24, –30, –36,
      C. 2                    D. 3                                  42,...... If sum of the first n terms of the sequence
      E. None of the above.                                         is 132, then the value of n is ___?
94.   In a triangle ABC, AB = 3, BC = 4 and CA = 5.                 A. 11                       B. 22
      Point D is the midpoint of AB, point E is on the              C. 13                       D. 18
      segment AC and point F is on the segment BC.                  E. 24
      If AE = 1.5 and BF = 0.5 then ∠ DEF =                    Question No. 101 - 102 are followed by two
      A. 30°                 B. 60°                            statements labelled as I and II. You have to decide if
      C. 45°                 D. 75°
                                                               these statements are sufficient to conclusively
      E. Cannot be determined
                                                               answer the question. Choose the appropriate answer
                            1                                from options given below:
95.   If 3f ( x + 2 ) + 4f       = 4x,x ≠ –2 , then f(4) =
                            x +2                                   A .If Statement I alone is sufficient to answer the
      A. 7                        B. 52 / 7                             question.
      C. 8                        D. 56 / 7                          B. If Statement II alone is sufficient to answer the
      E. None of the above.                                             question.
96.   A train left station X at A hour B minutes. It reached         C. If Statement I and Statement II together are
      station Y at B hour C minutes on the same day,                    sufficient but neither of the two alone is
      after travelling for C hour A minutes (clock shows                sufficient to answer the question.
      time from 0 hours to 24 hours). Number of possible             D. If either Statement I or Statement II alone is
      value(s) of A is ___.
                                                                        sufficient to answer the question.
      A. 3                        B. 2
                                                                     E. Both Statement I and Statement II and
      C. 1                        D. 0
      E. None of the above.                                             insufficient to answer the question.

XAT – 2008                                                                                                                17
Xat 2008 questions
Xat 2008 questions
Xat 2008 questions

Mais conteúdo relacionado

Mais procurados

Emotional Intelligence Training
Emotional Intelligence TrainingEmotional Intelligence Training
Emotional Intelligence TrainingDavidGMontague
 
Overcoming Shame in Borderline Personality Disorder (BPD) Using Compassion Fo...
Overcoming Shame in Borderline Personality Disorder (BPD) Using Compassion Fo...Overcoming Shame in Borderline Personality Disorder (BPD) Using Compassion Fo...
Overcoming Shame in Borderline Personality Disorder (BPD) Using Compassion Fo...Kenneth Smith, M.S.
 
Emotional intelligence and Emotional Resilience in Social Work
Emotional intelligence and Emotional Resilience in Social WorkEmotional intelligence and Emotional Resilience in Social Work
Emotional intelligence and Emotional Resilience in Social WorkClaudia Megele
 
Compassion focused therapy - Justin La Rose
Compassion focused therapy - Justin La RoseCompassion focused therapy - Justin La Rose
Compassion focused therapy - Justin La RoseJustinLaRose3
 

Mais procurados (6)

Emotional Intelligence Training
Emotional Intelligence TrainingEmotional Intelligence Training
Emotional Intelligence Training
 
Overcoming Shame in Borderline Personality Disorder (BPD) Using Compassion Fo...
Overcoming Shame in Borderline Personality Disorder (BPD) Using Compassion Fo...Overcoming Shame in Borderline Personality Disorder (BPD) Using Compassion Fo...
Overcoming Shame in Borderline Personality Disorder (BPD) Using Compassion Fo...
 
Emotional intelligence and Emotional Resilience in Social Work
Emotional intelligence and Emotional Resilience in Social WorkEmotional intelligence and Emotional Resilience in Social Work
Emotional intelligence and Emotional Resilience in Social Work
 
Anger management
Anger managementAnger management
Anger management
 
Psychotherapy & talk therapy
Psychotherapy & talk therapyPsychotherapy & talk therapy
Psychotherapy & talk therapy
 
Compassion focused therapy - Justin La Rose
Compassion focused therapy - Justin La RoseCompassion focused therapy - Justin La Rose
Compassion focused therapy - Justin La Rose
 

Semelhante a Xat 2008 questions

Emotional intelligence for personal growth
Emotional intelligence for personal growthEmotional intelligence for personal growth
Emotional intelligence for personal growthPreeti Bhaskar
 
Emotional intelligence for personal growth
Emotional intelligence for personal growthEmotional intelligence for personal growth
Emotional intelligence for personal growthPreeti Bhaskar
 
Chapter 13 and 14 emotions, stress, and health
Chapter 13 and 14   emotions, stress, and healthChapter 13 and 14   emotions, stress, and health
Chapter 13 and 14 emotions, stress, and healthswenson_n111
 
Theory of emotions created by vinod sawant
Theory of emotions created by vinod sawantTheory of emotions created by vinod sawant
Theory of emotions created by vinod sawantkavshal sawant
 
The Emotionally Intelligent Practitioner: Past, Present and Implications for ...
The Emotionally Intelligent Practitioner: Past, Present and Implications for ...The Emotionally Intelligent Practitioner: Past, Present and Implications for ...
The Emotionally Intelligent Practitioner: Past, Present and Implications for ...Christina R. Wilson, Ph.D.
 
Ch. 12: Emotions, Stress, & Health
Ch. 12: Emotions, Stress, & HealthCh. 12: Emotions, Stress, & Health
Ch. 12: Emotions, Stress, & Healthkbolinsky
 
Consumer behavior week5_socialprocessses
Consumer behavior week5_socialprocesssesConsumer behavior week5_socialprocessses
Consumer behavior week5_socialprocesssesEthan Chazin MBA
 
2nd Assignment of organization Behavior, M.com
2nd Assignment of organization Behavior, M.com 2nd Assignment of organization Behavior, M.com
2nd Assignment of organization Behavior, M.com Danish Saqi
 
Rational emotive behavior therapy
Rational emotive behavior therapyRational emotive behavior therapy
Rational emotive behavior therapyNeha Bhansali
 
Session 15 emotional intelligence ( SMS )
Session 15 emotional intelligence ( SMS )Session 15 emotional intelligence ( SMS )
Session 15 emotional intelligence ( SMS )Amaan Hussain
 
Emotional Intelligence
Emotional Intelligence Emotional Intelligence
Emotional Intelligence Sidra Akhtar
 

Semelhante a Xat 2008 questions (20)

Theories Personality
Theories PersonalityTheories Personality
Theories Personality
 
Group 5 Report.pptx
Group 5 Report.pptxGroup 5 Report.pptx
Group 5 Report.pptx
 
Emotional intelligence for personal growth
Emotional intelligence for personal growthEmotional intelligence for personal growth
Emotional intelligence for personal growth
 
Emotional intelligence for personal growth
Emotional intelligence for personal growthEmotional intelligence for personal growth
Emotional intelligence for personal growth
 
PSY101 Week 6 emotions
PSY101 Week 6 emotionsPSY101 Week 6 emotions
PSY101 Week 6 emotions
 
Chapter 13 and 14 emotions, stress, and health
Chapter 13 and 14   emotions, stress, and healthChapter 13 and 14   emotions, stress, and health
Chapter 13 and 14 emotions, stress, and health
 
Psycho socio-emotion
Psycho socio-emotionPsycho socio-emotion
Psycho socio-emotion
 
ch 13 & 14 emotion, stress & health
ch 13 & 14 emotion, stress & healthch 13 & 14 emotion, stress & health
ch 13 & 14 emotion, stress & health
 
Theory of emotions created by vinod sawant
Theory of emotions created by vinod sawantTheory of emotions created by vinod sawant
Theory of emotions created by vinod sawant
 
The Emotionally Intelligent Practitioner: Past, Present and Implications for ...
The Emotionally Intelligent Practitioner: Past, Present and Implications for ...The Emotionally Intelligent Practitioner: Past, Present and Implications for ...
The Emotionally Intelligent Practitioner: Past, Present and Implications for ...
 
Ch. 12: Emotions, Stress, & Health
Ch. 12: Emotions, Stress, & HealthCh. 12: Emotions, Stress, & Health
Ch. 12: Emotions, Stress, & Health
 
Emotions
EmotionsEmotions
Emotions
 
EQ FUTUREPOINT
EQ FUTUREPOINTEQ FUTUREPOINT
EQ FUTUREPOINT
 
Consumer behavior week5_socialprocessses
Consumer behavior week5_socialprocesssesConsumer behavior week5_socialprocessses
Consumer behavior week5_socialprocessses
 
Emotion
EmotionEmotion
Emotion
 
2nd Assignment of organization Behavior, M.com
2nd Assignment of organization Behavior, M.com 2nd Assignment of organization Behavior, M.com
2nd Assignment of organization Behavior, M.com
 
Rational emotive behavior therapy
Rational emotive behavior therapyRational emotive behavior therapy
Rational emotive behavior therapy
 
edu
eduedu
edu
 
Session 15 emotional intelligence ( SMS )
Session 15 emotional intelligence ( SMS )Session 15 emotional intelligence ( SMS )
Session 15 emotional intelligence ( SMS )
 
Emotional Intelligence
Emotional Intelligence Emotional Intelligence
Emotional Intelligence
 

Xat 2008 questions

  • 1. XAT Question Paper 2008 Instructions : 1. This question paper consists of three sections A, B and C with 38, 38 and 44 questions respectively, i.e. a total 120 questions. Total time for the test is 120 minutes. 2. You are required to answer questions from all three sections and expected to maximize scores in each section. 3. Each correct answer will be awarded 1 (one) mark. NEGATIVE MARKS (one fourth of a mark) may be deducted for the first six incorrect answers in each section and 0.5 (half a mark) for each incorrect answer thereafter. SECTION A: VERBAL AND LOGICAL ABILITY Analyse the passage given and provide an buying more confident in buying, and much more willing appropriate answer for the question nos. 1 through to tolerate things like waiting in line. On the other hand, 6 that follow. being in a mood makes buying behaviour in the “right mood” by the use of music and friendly staff or, say, Every conscious mental state has a qualitative character opens bakeries in shopping malls that delight the passer- that we refer to as mood. We are always in a mood that by with the smell of fresh bread. is pleasurable or unpleasurable to some degree. It may Thayer views moods as a mixture of biological and be that bad moods relate to their being too positive psychological influences and, as such, a sort of clinical reinforcement in a person’s current life and too many thermometer, reflecting all the internal and external punishments. In any case, moods are distinguished from events that influence us. For Thayer, the key components emotions proper by not being tied to any specific object. of mood are energy and tension in different combinations. But, this distinction is not watertight, in that emotions A specific mixture of energy and tension, together with need not be directed at objects that are completely the thoughts they influence, produces moods. He discuss specific (we can be angry just at people generally) while four mood states: there is always a sense of a mood having a general objective like the state of the world at large. Moods • Calm - energy: he regards this as the optimal manifest themselves in positive or negative feelings that mood of feeling good are tied to health, personality, or perceived quality of • Calm - tiredness: he regards this as feeling little life. Moods can also relate to emotions proper as in the tired without any stress, which can be pleasant. aftermath of an emotional incident such as the failure to • Tense - energy: involves low level of anxiety secure a loan. A mood on this basis is the mind’s suited to a fight - or - flight disposition. judgment on the recent past. For Goldie, emotion can • Tense - tiredness: is mixture of fatigue and bubble up and down within a mood, while an emotion anxiety, which underlines the unpleasant feeling can involve characteristics that are non-object specific. of depression. What is important for marketing is that moods colour People generally can “feel down” or “feel good” as result outlook and bias judgements. Hence the importance of of happenings in the world around them. This represents consumer confidence surveys, as consumer confidence the national mood. People feel elated when the national typically reflects national mood. There is mood- soccer team wins an international match or depressed congruence when thoughts and actions fall in-line with when their team has lost. An elated mood of calm -energy mood. As Goleman says, there is a “constant stream of is an optimistic mood, which is good for business. feeling” that runs “in perfect to our steam of thought”. Consumers, as socially involved individuals, are deeply Mood congruence occurs because a positive mood influenced by the prevailing social climate. Marketers evokes pleasant associations that lighten subsequent recognize the phenomenon and talk about the national appraisals (thoughts) and actions, while a negative mood being, say for or against conspicuous consumption. arouses future judgment and behavior. When consumers Moods do change, though. Writing early in the nineteenth are in a good mood, they are more optimistic about century. XAT – 2008 1
  • 2. Toqueville describers an American elite embarrassed by but not with action. the ostentation of material display; in the “Gilded Age”, E. When moods are synchronous with action but sixty years later, many were only too eager to embrace not with though. a materialistic vulgarity. The problem lies in anticipating 4. Implication and Proposition are defined as follows: changes in national mood, since a change in mood Implication: a statement which follows from the affects everything from buying of equities to the buying given text. of houses and washing machines. Thayer would argue Proposition: a statement which forms a part of that we should be interested in national events that are the given text. likely to produce a move towards a tense - tiredness Consider the two statements below and decide state or toward a calm - energy state, since these are whether they are implications or propositions. the polar extremes and are more likely to influence I. The marketers should understand and make behavior. Artists sensitive to national moods express the use of moods and emotions in designing and long-term changes. An example is the long - tem selling products and services. emotional journey from Charles Dickens’ depiction of II. Consuming is nothing but way of feeling the the death of little Nell to Oscar Wilde’s cruel flippancy void in consciousness. about it. “One would have to have a heart of stone not to A. Both statements are implications. laugh at the death of little Nell”. Which reflects the mood B. First is implication, second is proposition. change from high Victorian sentimentality to the acerbic C. Both are propositions. cynicism of the end of the century, as shown in writers D. First is propositions, second is implication. like Thomas Hardy and artists like Aubrey Beardsley. E. Both are neither implication nor proposition. Whenever the mind is not fully absorbed, consciousness is no longer focused and ordered. Under such conditions 5. Which statements from the ones given below are the mind falls into dwelling on the unpleasant, with a correct? negative mood developing. 1. In general, emotions are object specify 2. In general, moods are not object specific. Csikszentmihalyi argues that humans need to keep 3. Moods and emotions are same. consciousness fully active is what influences a good deal 4. As per Thayer, moods are a mix of biological of consumer behaviour. Sometimes it does not matter and psychological influences what we are shopping for - the point is to shop for A. 1, 2, 3 B. 2, 3, 4 anything, regardless, as consuming is one way to respond C. 1, 2, 4 D. 2, 4, 3 to the void in consciousness when there is nothing else E. All four are right to do. 6. The statement “Moods provide energy for human 1. Which one of the following statements best actions” is_____. summarizes the above passage? A. always right. B. sometimes right. A. The passage highlights how moods affect nations. C. always wrong B. The passage draws distinction between moods D. not derived from the passage. and emotions. E. contradictory. C. Some writer influenced national moods Directions (7 to 9): Carefully read the statements in through their writings. the questions below and arrange them in a logical D. Thayer categorised moods into four sates. E. The passage highlights the importance of order. moods and emotions in marketing. 7: 1. So too it is impossible for there to be proposition 2. Which of the following is the closest to of ethics. Proposition cannot express that is “conspicuous consumption” in the passage? higher. A. Audible consumption 2. The sense of the world must be outside the B. Consumption of material items for impressing world. In the world everything is as it is and others everything happens as it does happen: in it no C. Consumption driven by moods and emotions value exists-and if it did exist, it would have no D. Socially responsible consumption value. If there is any value that does have a E. Private but not public consumption value, it must lie outside whole sphere of what 3. What is “moods congruence”? happens and is the case. For all that happens A. When moods and emotions are synchronized. and is the case is accidental. What makes it B. When emotions are synchronous with actions non-accidental cannot lie within the world, since and thoughts. if it did it would it self be accidental. It must lie C. When moods are synchronous with thoughts outside world. and actions. D. When moods are synchronous with thoughts 2 XAT – 2008
  • 3. 3. It is clear that ethics cannot be put into words. A. ambivalence, prostate, immoral, amoral Ethics is transcendental. B. ambivalence, prostrate, amoral, immoral 4. All propositions are of equal value. C. ambiguity, prostrate, amoral immoral A. 4 - 2 - 1 - 3 B. 2 - 1 - 3 - 4 D. ambivalence, prostrate, immoral, amoral C. 1 - 3 - 4 - 2 D. 4 - 3 - 1 - 2 E. ambiguity, prostrate, immoral amoral E. 3 - 1 - 2 - 4 12. It is not ________ democratic that the parliament 8: 1. The fact all contribute only to setting the should be _________ on issues and resort to problem, not to its solution. passing _______ rather than have an open debate 2. How things are in the world is a matter of on the floor of the house. complete indifference for what is higher. God A. quite, quite, ordinances does not reveal himself in the world. B. quite, quiet, ordnances 3. To view the world sub specie aeterni is to view C. quiet, quite, ordnances it as a whole - a limited whole. D. quiet, quiet, ordinances Feelings the world as a limited whole - it is this E. quite, quiet, ordinances that is mystical. 4. It is not how things are in the world that is 13. In a case of acute ______, _______ membranes mystical, but that it exists. secrete excessive __________. A. 1 - 2 - 3 - 4 B. 2 - 1 - 3- 4 A. sinus, mucous, mucous C. 3 - 1 - 4 - 2 D. 3 - 4 - 1 - 2 B. sinus, mucus, mucous E. 2 - 1 - 4 - 3 C. sinusitis, mucous, mucus D. sinus, mucous, mucus 9: 1. The operation is what has to be done to one E. sinusitis, mucus, mucous proposition in order to make other out of it. 2. Structure of proposition stands in internal 14. If a person makes the statement: “ I never speak relations to one another. the truth.” The person can be said to be 3. In order to give prominence to these internal ________. relations we can adopt the following mode of A. speaking the truth. expression: we can represent a proposition as B. lying. the result of an operation that produces it out C. lying as well as speaking the truth of other propositions (which are bases of the D. making a logically contradictory statement operation). E. partially speaking the truth and partially lying. 4. An operation is the expression of a relation Analyse the passage given and provide an between the structure of its result and of its appropriate answer for the question nos. 15 through bases. A. 2 - 3 - 4 - 1 B. 1 - 2 - 3 - 4 18 that follow. C. 4 - 3 - 1 - 2 D. 2 - 1 - 3 - 4 Enunciated by Jung as an integral part of his psychology E. 4 - 1 - 2 - 3 in 1916 immediately after his unsettling confrontation Directions (10 to 14): Choose the appropriate words with the unconscious, the transcendent function was seen to fill in the blanks. by Jung as uniting the opposites, transforming psyche, and central to the individuation process. 10. Mark Twain was responsible for many striking, mostly cynical ____, such as “ Always do right. It also undoubtedly reflects his personal experience in That will gratify some of the people, and astonish coming to terms with the unconscious. Jung portrayed the rest”. ________ can sometimes end up as the transcendent function as operating through symbol _____, but rarely would someone use them as and fantasy and mediating between the opposites of an________. consciousness and the unconscious to prompt the A. epitaphs, Epitaphs, epigrams, epigraph emergence of a new, third posture that transcends the B. epigraphs, Epigraphs, epitaphs, epigraph C. epigrams, Epigrams, epigraphs, epitaph two. In exploring the details of the transcendent function D. epigrams, Epitaphs, epigrams, epigraph and its connection to other Jungian constructs, this work E. epitaphs, Epitaphs, epigraphs, epigram has unearthed significant changes, ambiguities, and inconsistencies in Jung’s writings. Further, it has identified 11. A candidate in the medical viva voce exam faced two separate images of the transcendent function: (1) a tinge of intellectual ______ when asked to spell the ___ gland. The fact that he carried notes on the narrow transcendent function, the function or process his person would definitely be termed as ___ by within Jung’s pantheon of psychic structures, generally factually, but may be termed as _______ by more seen as the uniting of the opposites of consciousness generous sections of students. and the unconscious from which a new attitude emerges; XAT – 2008 3
  • 4. and (2) the expansive transcendent function, the root Directions: (19 to 26): Go through the caselets below metaphor for psyche or being psychological that and answer the questions that follow. subsumes Jung’ pantheon and that apprehends the most fundamental psychic activity of interacting with the Question No. (19 to 20): According to recent reports, unknown or other. This book has also posited that the CEOs of large organisations are paid more that CEOs expansive transcendent function, as the root metaphor of small organizations. It does not seem fair that just for exchanges between conscious and the unconscious, because a CEO is heading a big organization s/he should is the wellspring from whence flows other key Jungian be paid more. CEOs salary should be related to structures such as the archetypes and the Self, and is performance, especially growth in terms of sales and the core of the individuation process. The expansive profits. Of course, big organisations are more complex transcendent function has been explored further by than the small, but all CEOs require significant amount of energy and time in managing organisations. There is surveying other schools of psychology, with both death no proof then CEOs of small organisations. All CEOs and non-death orientations, and evaluating the should be paid according to their performance. transcendent function alongside structures or process in those other schools which play similar mediatory and 19. A person seeking to refute the argument might / or transitional roles. argue that A. CEOs should be paid equally. 15. The above passage is most likely an excerpt from: B. Managing big organisation is more challenging A. research note that small . B. An entry on a psychopathology blog C. If CEOs of small companies perform well, the C. A popular magazine article company would become big and so would be D. A newspapers article CEOs salary. E. A scholarly treatise D. CEOs who travel more should be paid more. E. Highly qualified CEOs should be paid more 16. It can be definitely inferred from the passage because they have acquired difficult education. above that A. The expansive transcendent function would 20. Which of the following, if true, would strengthen include elements of both the Consciousness the speaker’s argument? and the Unconscious. A. CEOs of small organisations come from good B. The transcendent is the core of the individuation educational background. process. B. CEOs in big organisation take much longer to C. Archetypes emerge from the narrow reach top, as compared to their counterparts in transcendent function. small organisations. D. The whole work, from which this excerpt is C. CEOs of big organisations are very difficult to taken, primarily concerns itself with the hire. inconsistencies is Jung’s writings. D. A few big family businesses have CEOs from E. Jung’s pantheon of concept subsumes the root within the family. metaphor of psyche. E. Big organisations contribute more towards moral development of society. 17. A comparison similar to the distinction between the two images of the transcendent function would Question no (21 to 22) Hindi ought to be the official be: language of India. There is no reason of the government A. raucous: hilarious B. synchronicity: ontology to spend money printing documents in difference C. recession: withdrawal D. penurious: decrepit languages, just to cater to people who cannot read/write E. None of these Hindi. The government has better ways to spend tax payers’ money. People across India should read/write 18. As per the passage, the key Jungian structure - Hindi or learn it at the earliest. other that the Self - that emerges from the expansive transcendent function may NOT be 21. Which of the following, if true, would weaken the expressed as a(n): speaker’s argument the most? A. Stereotype B. Anomaly A. The government currently translates official C. Idealized model D. Original pattern documents into more than eighteen languages. E. Epitome B. Hindi is the most difficult language in the world to speak. 4 XAT – 2008
  • 5. C. People who are multilingual usually pay 24. According to a recent research conducted by the maximum taxes. district road planning department, ten percent D. Mot people who travel across India learn Hindi students come with parents in cars, twenty percent within five years. students use auto - rickshaws, twenty percent E. Making Hindi the official language is a students use taxis, forty percent students use the politically unpopular idea. school buses and ten percent students live in the hostel inside the school. 22. United Nations members contribute funds, Which of the following is true about the above proportionate to their population, for facilitating paragraph? smooth functioning of the UN. By 2010, India, A. It extends speaker’s argument using a analogy. being the most populous nation on the planet, B. It extends the speaker’s argument using statistical data. would contribute the maximum amount to the UN. C. It is similar to speaker’s argument. Therefore, official language of United Nations D. It contradicts the speaker’s argument using should be changed to Hindi.? statistical data. Which of the following is true? E. It concludes speaker’s argument by using A. The point above contradicts the speaker’s personalization. argument. B. The point above is similar to speaker’s Question No (25 and 26): History, if viewed as a argument. repository not merely of anecdotes or chronology, could C. The point above concludes speaker ’s produce a decisive transformation in the image of argument. science by which we are now possessed. That image D. The point above extends the speaker ’s has previously been drawn, even by scientists argument. themselves, mainly from the study of finished scientific E. The point above strengthens the speaker’s achievements as these are recorded in the classics and, argument. more recently, in the textbooks from which each new scientific generation learns to practice its trade. Questions No (23 and 24): The Bistupur - Sakchi corner needs a speed - breaker. Loyola school children cross 25. Which of the following best summarizes the above this intersection, on their way to the school, and many a paragraph? times do not check out for traffic. I get to read regular A. Scientific achievements are record in classics reports of cars and other vehicles hitting children. I know and text books. that speed - breakers are irritating for drivers and I know B. Different ways of looking at history can that children cannot be protected from every danger, produce altogether different knowledge. but this is one of the worst intersections in town. There C. History of science can be inferred from needs to be a speed - breaker so that vehicles have to finished scientific achievement slow down and the children be made safer. D. Text books may be biased. E. All of above. 23. Which of the following arguments is used in the 26. Which of the following statements is the author above passage? most likely to agree with? A. Analogy - comparing the intersection to A. History of science presents a scientific way of something dangerous looking at scientific development and thus B. Statistical analysis - noting the number of contributes to progress in science. children hit by vehicles B. History of science should contain only the C. Personalization - telling the story of one child’s chronology of the scientific achievements. near accident at the intersection C. More number of scientific theories results in D. Attack - pointing out people who are against more number of publications, which benefits speed - breakers as being uncaring about publishers. children D. History of science should purposely present E. Emotive - referring to the safety of children to different images of science to people. get people interested E. History of science can present multiple interpretations to people regarding the process of scientific development. XAT – 2008 5
  • 6. Direction (27 and 28): Go through the caselets below not a single entity but an amalgamation, a “construct” and answer the questions that follow. arising from the conjoining of innumerable, discrete parts. Modern scholarship has, quite properly, tended to explore 27. Goodricke Group Ltd is planning to give top priority these elements in isolation. (In part, this trend represents to core competence of production and marketing the conscious reversal of the stance taken by an earlier of tea in 2007. The company intends to increase generation of scholars whose work reified India into a the production of orthodox varieties of tea. monolithic entity - a critical element in the much maligned Goodricke is planning to invest Rs. 10 crore to “Orientalist” enterprise). Nonetheless, the representation modernise the factories. The company has of India as a singular “Whole” is not an entirely capricious announced a net profit of Rs. 5.49 crore for 2006 enterprise; for India is an identifiable entity, united by - if as against Rs. 3.76 crore in 2005. not born out of - certain deep and pervasive structures. Which of the following can be deduced from the Thus, for example, the Hindu tradition has long caselet? maintained a body of mythology that weaves the A. Production and marketing is core competence disparate temples, gods, even geographic landscapes of Goodricke Group. that exist throughout the subcontinent into a unified, albeit B. Increase in production of existing products syncretic, whole. enhances core competence. C. Core competence leads to modernization. In the realm of thought there is no more pervasive, D. Goodricke has given top priority to production unifying structure than karma. It is the “doctrine” or “law” because it has earned net profit of Rs. 5.49 that ties actions to results and creates a determinant crore. link between an individual’s status in this life and his or E. Core competence can be used for furthering her fate in future lives. Following what is considered to company’s interests. be its appearances in the Upanishads, the doctrine reaches into nearly every corner of Hindu thought. 28. The author reflects on the concept of Blue Ocean Indeed, its dominance is such in the Hindu world view Strategy. He explains that this concept delivers that karma encompasses, at the same time, life-affirming an instinctive framework for developing and life-negating functions; for just as it defines the world uncontested market space and making the in terms of the “positive” function of delineating a doctrine competition irrelevant. The author remarks that of rewards and punishments, so too it defines the world the Blue Ocean Strategy is about having the best through its “negative” representation of action as an all mix of attributes that result in creation of but inescapable trap, an unremitting cycle of death and uncontested market space and high growth, and rebirth. not about being the best. The above paragraph appears to be an attempt Despite - or perhaps because of - karma’s ubiquity, the at doctrine is not easily defined. Wendy Doniger O’ Flaherty A. reviewing an article or a book on Blue Ocean reports of a scholarly conference devoted to the study strategy. of karma that although the participants admitted to a B. defining Blue Ocean strategy. general sense of the doctrine’s parameters, considerable C. developing the framework for Blue Ocean item was in a “lively but ultimately vain attempt to define strategy. ... karma and rebirth”. The base meaning of the term D. highlighting how Blue Ocean strategy leads “Karma” (or, more precisely, in its Sanskrit stem form, to better returns. Karman a neuter substantive) is “action”. As a doctrine, E. None of above. Karma encompasses a number of quasi - independent concepts: rebirth (punarjanam), consequence (phala, Analyse the following passage and provide an literally “fruit,” a term that suggest the “ripening” of actions appropriate answer for the question nos. 29 through into consequences), and the valuation or “ethic-ization” 35 that follow. of acts, qualifying them as either “good” (punya or sukarman) or “bad” (papam or duskarman). India is renowned for its diversity. Dissimilitude abounds in every sphere from the physical elements of its land In a general way, however, for at least the past two and people to the intangible workings of its beliefs and thousand years, the following (from the well known text, practices. Indeed, given this variety, India appears to be the Bhagvata Purana) has held true as representing the 6 XAT – 2008
  • 7. principal elements of the Karma doctrine: “The same 34. As per the author, which of the following person enjoys the fruit of the same sinful or a meritorious statements is wrong? act in the next world in the same manner and to the A. India is a diverse country. manner and extent according to the manner and extent B. Doctrine of Karma runs across divergent Hindu to which that (sinful or meritorious) act has been done thoughts. by him in this world.” Nevertheless, depending on the C. Doctrine of Karma has a rich scholarly doctrine’s context, which itself ranges from its discourse. appearance in a vast number of literary sources to its D. Modern scholars have studied Hinduism as a usage on the popular level, not all these elements may syncretic whole. be present (though in a general way they may be implicit). E. Scholars could not resolve the meaning of Karma. 29. The orientalist perspective, according to the author: 35. Which of the following, if true, would be required A. Viewed India as country of diversity. for the concept of Karma - as defined in Bhagavata B. Viewed India both as single and diverse entity. Purana - to be made equally valued across C. Viewed India as if it was a single and unitary different space-time combinations? entity devoid of diversity. A. The information of the past actions and the D. Viewed India as land of ?Karma. righteousness of each action would be E. Viewed India in the entirely. embodied in the individual. 30. “Reify” in the passage means: B. Karma is judged based on the observers’ A. Reversal of stance perception, and hence the observer is a B. Unitary whole necessary condition for its validity. C. To make real out of abstract C. Karma is an orientalist concept limited to D. Diversity oriental countries. E. Unity in diversity D. Each epoch will have its own understanding of Karma and therefore there can not be uniform 31. “Ethic-ization” in the passage means validity of the concept of Karma. A. Process making something ethical E. Each space-time combination would have B. Converting unethical persons into ethical different norms of righteousness and their C. Teaching ethics respective expert panels which will judge each D. Judging and evaluation action as per those norms. E. None of these 32. Consider the following statements: Analyse the passage given and provide an 1. Meaning of Karma is contextual. appropriate answer for the question nos. 36 through 2. Meaning of Karma is not unanimous. 38 that follow. 3. Meaning of Karma includes many other quasi independent concepts. Deborah Mayo is a philosopher of science who has 4. Karma also means actions and their rewards. attempted to capture the implications of the new experimentalism in a philosophically rigorous way. Mayo Which of the above statements are true? focuses on the detailed way in which claims are validated A. 1, 2, 3 B. 2, 3, 4 by experiment, and is concerned with identifying just C. 1, 3, 4 D. None of these what claims are borne out and how. A key idea underlying E. All the four are true. her treatment is that a claim can only be said to be supported by experiment if the various ways in which 33. The base meaning of “Karma, is: the claim could be at fault have been investigated and A. Reward and punishment. eliminated. A claim can only be said to be borne out by B. Only those actions which yield a “phala”. experiment and a severe test of a claim, as usefully C. Any action. construed by Mayo, must be such that the claim would D. Ripening of actions into consequences. be unlikely to pass it if it were false. E. None of these. Her idea can be explained by some simple examples. Suppose Snell’s law of refraction of light is tested by XAT – 2008 7
  • 8. some very rough experiments in which very large B. Precise measurement is both a necessary and margins of error are attributed to the measurements of sufficient condition to ensure validity of angels of incidence and refractions, and suppose that conclusion resulting from an experiment. the results are shown to be compatible with the law within C. Experimental data might support multiple those margins of error. Has the law been supported by theoretical explanations at the same time, experiments that have severely tasted it? From Mayo’s hence validity of theories needs to be tested [perspective the answer is “no” because, owing to the further. roughness of the measurements, the law of refraction D. Precise measurement along with would be quite likely to pass this test even if it were experimenter’s knowledge of the theory false and some other law differing not too much from underpinning the experiment is sufficient to Snell’s law true. An exercise I carried out in my school- ensure the validity of conclusions drawn from teaching days serves to drive this point home. My experiment. students had conducted some not very careful E. All of these experiments to test Snell’s law. I then presented them with what had been suggested in antiquity and mediaeval 37. As per Mayo’s perspective, which of the following times, prior to the discovery of Snell’s law and invited best defines the phrase “scientific explanation”? the students to test them with the measurements they A. One which is most detailed in its explanation had used to test Snell’s law; because of the wide margins of natural phenomena. of error they had attributed to their measurements, all of B. One which survives examinations better than these alternative laws pass the test. This clearly brings other explanations. out the point that the experiments in question did not C. One which has been thoroughly tested by constitute a severe test of Snell’s law. The law would scientific experts. have passed the test even if it were false and one of the D. One which refutes other explanations historical alternatives true. convincingly. E. All of these. 36. Which of the following conclusion can be drawn from the passage? 38. The author’s use of Snell’s law of refractions to A. Precise measurement is a sufficient condition illustrate Mayo’s perspective can best said to be to ensure validity of conclusions resulting from A. Contrived. B. Premeditate. an experiment. C. Superfluous. D. Illustrative E. Inadequate. SECTION B: ANALYTICAL REASONING & DECISION MAKING Directions (Questions No. 39 to 59): Each group of schedule MUST satisfy the following restrictions: questions in this section is based on a set of conditions. In answering some of the questions, it She conducts quizzes on exactly three mornings. may be useful to draw a rough diagram. Choose the If she conducts quizzes on Monday, she does not conduct response that most accurately and completely a quiz on Tuesday. answers each question and blacken the corresponding space on your answer sheet. She lectures in the afternoon on exactly two consecutive calendar days. Questions No 39 42: Professor Mukhopadhay works only on Mondays, Tuesdays, Wednesdays, Fridays and She evaluates quizzes on exactly one morning and three Saturdays. She performs four different activities- afternoons. lecturing, conducting quizzes, evaluating quizzes and working on consultancy projects. Each working day she She works on consultancy project on exactly one performs exactly one activity in the morning and exactly morning. one activity in the afternoon. During each week her work On Saturday, she neither lectures nor conducts quizzes. 8 XAT – 2008
  • 9. 39. On Wednesdays, the Professor could be medical institutes, two management institutes, and two scheduled to technical institutes. These eight institutions are located A. Conduct a quiz in the morning and lecture in as follows: the afternoon. B. Work on a consultancy project in the morning No institution is in more than one of the states. and conduct a quiz in the afternoon. None of the states contains more than one management C. Lecture in the morning and evaluate quizzes institute, and none contains more than one technical in the afternoon. institute. D. Conduct a quiz in the morning and work on None of the states contains both a management institute consultancy project in the afternoon. and a technical institute. E. Evaluate quizzes in the morning and evaluate Each management institute is located in a state that quizzes in the afternoons. contains at least one medical institute. The technical institutes are located in two states that do 40. Which one of the following statements must be not share a common boundary. true? State 3 contains a technical institute, and State 6 contains A. There is one day on which she evaluates a management institute. quizzes both in the morning and in the afternoon. 43. Which one of the following could be true? B. She works on consultancy project on one of A. State 1 contains exactly one technical institute the days on which lecturers. B. State 2 contains exactly one management C. She lectures on one of the days on which she institute conducts quiz. C. State 5 contains exactly one technical institute D. She works on consultancy project on one of D. State 6 contains exactly one technical institute the days on which she evaluates quizzes. E. State 1 contains exactly one medical institute E. She lecturers on one of the days on which evaluates quizzes. 44. A complete and accurate list of the states, any one of which could contain the management 41. If the Professor conducts a quiz on Tuesday, then institute that is not in State 6, would be ________. her schedule for evaluating quizzes could be A. 1, 4 B. 2, 4 A. Monday morning, Monday afternoon, Friday C. 1, 4, 5 D. 4, 5 morning, Friday afternoon. E. 1, 2, 4, 5 B. Monday morning, Friday afternoon, Saturday morning, Saturday afternoon 45. If each of the six states contains at least one of C. Monday afternoon, Wednesday morning, the eight institutions, then which one of the Wednesday afternoon, Saturday afternoon following must be true? D. Wednesday morning, Wednesday afternoon, A. There is management institute in state 1 Friday afternoon, Saturday afternoon B. There is a medical institute in state 2 E. Wednesday afternoon, Friday afternoon, C. There is a medical institute in state 3 Saturday morning, Saturday afternoon D. There is a medical institute in state 4 E. There is a management institute in state 4 42. Which one of the following must be a day on which Professor lectures? 46. If one of the states contains exactly two medical A. Monday B. Wednesday institutes and exactly one technical institute, then C. Friday D. Tuesday which combinational of three states might contain E. Saturday no medical institute? A. 1, 4, 5 B. 2, 3, 5 Question No 43 to 46: Six square states having equal C. 2, 4, 6 D. 1, 3, 5 area in a country are located in North-South directions E. 4, 5, 6 in two columns next to each other. States are located in the given order, State 1, State 3, and State 5 are on the western side and State 2, State 4, and State 6 are on the eastern side. Within the six states, there are exactly four XAT – 2008 9
  • 10. Question No 47 to 50: During a four-week period each of exactly six words and satisfies the following conditions: one of seven previously unadvertised products - G, H J, The six words are written from left to right on a single K, L, M and O - will be advertised,. A different pair of line in alphabetical order. these products will be advertised each week. Exactly The sentence can start with any word, and successive one of the products will be a members of two of these word is formed by applying exactly one of three four pairs. None of the other products gets repeated in operations to the preceding world: delete one letter; add any pair. a letter; replace a one letter with another. Further , the following constraints must be observed: At the most three of the six words can begin with the J is not advertised during a given week unless H is same letter. advertised during the immediately preceding week. Except for the first word each word is formed by a The product that is advertised twice is advertised during different operation used for the preceding word. week 4 but is not advertised during week 3. G is not advertised during a given week unless either J 51. Which one of the following could be a sentence or O is also advertised that week. in the word game? K is advertised during one of the first two weeks. A. Bzaeak blaeak laeak paeak paea paean O is one of the products advertised during week 3. B. Doteam goleam golean olean omean omman C. Crobek croeek roeek soeek sxoeek xoeek 47. Which one of the following could be the schedule D. Feted freted reted seted seteg aseteg of the advertisements? E. Forod forol forols forpls orpls morpls A. Week 1: G, J; week 2 : K, L; week 3: O, M; week 4: H, L 52. The last letter of the English alphabet that the B. Week 1: H, K; week 2: J, G; week 3: O, L; first word of a sentence in the word game can week 4: M, K begin with is C. Week 1: H, K; week 2: J, M; week 3: O, L; A. t B. w week 4: G, M C. x D. y D. Week 1: H, L; week 2: J, M; week 3: O, G; E. z week 4: K, L E. Week 1: K, M; week 2: H, J; week 3: O, G; 53. If the first word in a sentence is “illicit” and the week 4: L, M fourth word is “licit”, then the third word can be 48. If L is the product that is advertised during two of A. Implicit B. Explicit the weeks, which one of the following is a product C. Enlist D. Inlist that MUST be advertised during one of the weeks E. Elicit in which L is advertised? A. G B. H 54. If “clean” is the first word in a sentence and “learn” C. M D. K is another word in the sentence, then which one E. J of the following is a complete and accurate list of the positions “learn” could occupy? 49. Which one of the following is a product that could A. Third be advertised in any of the four weeks? B. Second, third, fourth A. H B. L C. Third, fourth, fifth, sixth C. K D. L D. Third, fourth E. O E. Third, fourth, fifth 50. Which one of the following is a pair of products 55. If the first word in a sentence consists of five that could be advertisement during the same letters, then the maximum number of letters that week? the fifth word in the sentence could contain is A. M and O B. G and H A. Four B. Five C. H and J D. H and O C. Six D. Seven E. K and O E. Eight Questions No 51 to 55: In a game, “words” (meaningful Question No 56 to 59: There are exactly ten stores and or meaningless) consist of any combination of at least no other building on a straight street in Bistupur Market. five letters of the English alphabets. A “sentence” consists On the northern side of the street, from West to East, 10 XAT – 2008
  • 11. are stores 1, 3, 5, 7 and 9; on the southern side of the Go through the situation and the accompanying street, also from West to east, are stores 2, 4, 6, 8 and table, and pick up the best alternatively to answer 10. The stores on the northern side are located directly Question nos. 60 to 62 across the street from those on the southern side, facing Question No 60-61: There are five sets of digits - Set each other in pairs, as follows: 1 and 2; 3 and 4; 5 and 6; A, Set B, Set C, Set D, and Set E as shown in given 7 and 8; 9 and 10. Each store is decorated with lights in diagram. Set A contains one digit, Set B contains two digits, Set C contains three digits, set D contains two exactly one of the following colours: green, red, and digits, and Set E contains one digit. yellow. The stores have been decorated with lights Rearrange the digits, across the sets, such that the according to the following conditions: number formed out of digits of Set C is multiple of the No store is decorated with lights of the same colour as numbers formed from digits in sets on either side. For those of any store adjacent to it. example; in the given diagram, SET C is a multiple of No store is decorated with lights of the same colour as digits in SET A and SET B but not of SET D and SET E those of the store directly cross the street from it. Yellow lights decorate exactly one store on each side of SET A SET B SET C SET D SET E the street. 7 28 196 34 5 Red lights decorate store 4. Yellow lights decorate store 5. 60. What is the minimum number of rearrangements 56. Which one of the following could be an accurate required to arrive at the solution? A rearrangement list of the colours of the lights that decorate stores is defined as an exchange of positions between 2, 4, 6, 8 and 10, respectively? digits across two sets. For example: when 1 from A. green, red, green, red, green set C is exchanged with 5 of set E, it is counted B. green, red, green, yellow, red as one rearrangement. C. green, red, yellow, red, green A. 2 B. 5 D. yellow, green, red, green, red C. 8 D. 3 E. yellow, red, green, red, yellow E. 7 57. If green lights decorate store 7, then each of the 61. Which of the following pair of digits would occupy following statements could be false set A and E? EXCEPT: A. 2 and 4 B. 2 and 6 A. Green lights decorate store 2 B. Green lights decorate store 10 C. 3 and 6 D. 3 and 9 C. Red lights decorate store 8 E. 4 and 8 D. Yellow lights decorate store 2 Question No. 62: Magic Box E. Red lights decorate store 9 5 15 1 16 58. Which one of the following statements MUST be 10 4 8 9 true? 11 6 12 2 A. Red lights decorate store 1 4 3 13 7 B. Green lights decorate store 10 Cut the square given above into four pieces along the C. Red lights decorate store 8 lines and rearrange in such a manner that sum of all D. Yellow lights decorate store 8 rows, columns and diagonals is equal to 34. One of the E. Yellow lights decorate store 10 pieces, comprising 1 and 8, is shown in the diagram given below 59. Suppose that yellow lights decorate exactly two stores on the south side of the street and exactly 1 one store on the north side. If all other conditions 8 remain the same, then which one of the following statements Must be true? A. Green lights decorate store1 How many numbers would be there in the largest B. Red lights decorate store 7 piece? C. Red lights decorate store 10 A. 5 B. 6 D. Yellow lights decorate store 2 C. 9 D. 10 E. Yellow light decorate store 8 E. 8 XAT – 2008 11
  • 12. Read the following situation and choose the best B. Reduce the number of female employees, as possible alternative. it is a business requirement. She should not let anything affect her business. Question No. 63: The surnames of four professionals C. Let the status quo continue. are: Bannerji, Chatterji, Mukherji and Pestonji. Their D. Henceforth hire only male employees. professions are accountant, lawyer, dentist and doctor E. She should close the business. (not necessarily in this order). The accountant and lawyer work in their offices, while the dentist and doctor work in 65. You, a recruitment manager, are interviewing their nursing homes. The accountant looks after Mayank, a hard- working young man, who has Mukherji’s and Chatterji’s account. Chatterji, dose not problems in speaking fluent English. He has know Bannerji, although his nursing home is in the same studied in vernacular medium schools and street as Bannerji’s office. Chatterji is not a doctor. colleges. Amongst the following options, what What are the occupations of the four people. would you choose to do, if your company has vacancies? A. Bannerji- Doctor, Chatterji- Dentist,Mukherji- A. I would hire him at all costs. accountant and Pestonji- Lawyer. B. I would hire him for the job he is good at, and B. Bannerji- Lawyer, Chatterji- Dentist, Mukherji- provide training in other areas. accountant and Pestonji- Doctor. C. I would hire him for production or finance job C. Bannerji- Lawyer, Chatterji- Dentist, Mukherji- but not for marketing job, which requires good Doctor and Pestonji- Accountant. communication skills. D. Bannerji- Doctor, Chatterji- Accountant, D. I would ask him to improve his communication Mukherji- Dentist and Pestonji- Lawyer. skills and come back again. E. Bannerji- Dentist, Chatterji- Lawyer, Mukherji- E. I would not hire him as he might be a burden Doctor and Pestonji- Accountant. on organisation because of his poor communication skills. Questions 64 - 67: Read the following situations and choose the best possible alternative. 66. The city of Nagar has a population of 10 million, 2 millions amongst whom are rich, 3 million poor 64. Seema was a finance manager in an MNC and and 5 million belong to the middle class. felt that gender discrimination at the workplace Saundarya Cosmetics manufactured and sold hampered her career growth. Frustrated, she quit beauty product to the rich class at a premium the job and started a company. While starting her price. Its products were very popular with company, Seema decided that she would have customers. Many people from the middle and poor equal proportion of males and females. Over the segments of the population aspired to buy these next six years, Seema emerged as a very products but could not afford because of the high successful entrepreneur and expanded her prices. Of late, sales growth has been stagnating business to eight locations in the country. in the rich segment. Which of the following is the However, Seema recently started facing an ethical best option for Saundarya Cosmetics to maximize dilemma because she realized that female long-term profits? employees were not willing to travel across cities A. Sell the same products at lower prices to middle and work late hours, as the work required them to and poor classes. do. Male employees did not hesitate undertaking B. Sell similar products, of different quality such work. Seema started to feel the pressure of standards with different brand names, to middle reducing the proportion of female employees. On classes and poor classes. the other hand, she is aware that equal C. Sell its products under different brand names representation was one of the strongest reasons to middle and poor classes. for her to have founded the company. What should D. Continue to target rich only and hope that she do as a conscientious female entrepreneur? today’s middle class would be tomorrow’s rich class. A. See if unwilling female employees could be E. Target middle class as it is the largest segment given assignments which do not require travel and forget about the rich. and involve less overtime. 12 XAT – 2008
  • 13. 67. A database software manufacturing company A. Apologize and fix up the bug for all customers found out that a product it has launched recently even if it has to incur losses. had a few bugs. The product has already been B. Do not tell customers about bugs and remove bought by more than a million customers. The only when customers face problems, even if it means losses for the customers. company realized that bugs could cost its C. Keep silent and do nothing. customers significantly. However, if it informs the D. Keep silent but introduce and improved product customers about the bug, it feared losing that is bug-free at the earliest. credibility. What would be the most ethical option E. Take the product off the mark and apologize to for the company? customers. Go through the table that follows and pick up the best alternative to answer question no. 68 to 70. Name of the Average of batmen Number of Number of times Number of times batsman based on past times dismissed scores more than performance dismissed around average a century below 20 RD 40 20 70 3 ST 44 20 60 10 SG 41 25 50 10 VS 31 50 20 15 RU 28 55 25 12 YS 35 40 40 10 VV 35 35 50 5 MK 30 30 45 5 MT 36 45 30 10 MD 45 30 50 10 Teams A, B, C and D are participating in a cricket tournament. Team A has to pick up five batsmen out of the ten available. All batsmen have played 100 matches each in the past. Past data indicates that C beats A 8 out of 10 times. B beats A 5 out of 10 times and D beats A 1 out of 10 times. The conditions for series are likely to be normal and bowling strength of all teams is same. Manager of Team A, based on his past experience, feels that team should take high risk against stronger opponents and low risk against weaker opponents for maximizing chances of wining the game. The average scores of the top 5 batmen for each team playing in the tournament are: C (270); B (215); D (180) and A (215). 68. Team A would play the third match with B. Based on the statistics above, whom should the manager choose so that A has maximum chances of winning? A. RD, RU, MU,VS,YS B. RD, VS, MT, RU, YS C. RD, VV,SG,VS, MD D. ST, RD,MK, MD, SG E. SG, RU, YS, MK,VV 69. Team A is playing its first match with team C. Based on the statistics above, whom should the manger choose so that the team has maximum chances of winning? A. RD, ST, SG, MD,YS B. VS, YS, RU,MD, MT C. RD, ST, SG, VS,MD D. YS, RU, VS, MK, MD E. ST, VS, RU, MD, SG 70. Team A would play the second match with D. Based on the statistics above, whom should the manger choose so that A has maximum chances of winning? A. RD, ST, MD,VS,YS B. ST, RD, VV, SG, MD C. RD, ST, SG, VS,MD D. SG, RU, YS, MK, MD E. ST, RD, MK,MD,SG XAT – 2008 13
  • 14. Read the following caselet and choose the best 71. The most important causal factor for this entire alternative (Question 71 - 76): episode could be: A. Trying to follow a divide-and-rule policy in his Mr. Rajiv Singhal, Chairman of the Board of Directors of dealings with the supervisors. Loha India Ltd., (a steel manufacturing company) had B. Inconsistent dealings of Mr. Thakur with just been visited by several other directors of the supervisors. company. The directors were upset with recent actions C. Paternalistic approach towards mature of the company president, Mr. Ganesh Thakur. They individuals in the organisation. demanded that the board consider firing the president. D. Legalistic approach to employee problems. E. Inadequate standards for measurement of Mr. Thakur, recently appointed as president, had supervisors’ on-job performance. undertaken to solve some of the management- employees problems by dealing directly with the 72. The situation with Mr.Lal could have been avoided individuals, as often as possible. The company did not if Mr. Thakur had have a history of strikes or any other form of collective 1. Delegated the task of negotiation of wage action and was considered to have good work culture. contracts for night shift employees to Personnel However, Mr. Thakur felt that by dealing directly with department. individuals, he could portray the management’s concern 2. Created a process for supervisors working the for the employees. An important initiative of Mr. Thakur night shift so that they could have and was to negotiate wages of the supervisors with each opportunity to interact with him. supervisor. In these negotiation meetings, he would not 3. Created an open door policy that would have involve anyone else, including the Personnel Department allowed employees to see him without any which reported to him, so as to take an unbiased decision. appointment. After negotiation, a wage contract would be drawn up 4. Postponed the decision of wage revision for for each supervisor. This, he felt, would recognize and supervisors in the night shift for two months, reward the better performers. Mr. Thakur successfully since supervisors were rotated on different implemented the process for most of the supervisors, shifts after every two months. except those working in night shift with that of supervisors The option that best arranges the above of the day shift. managerial interventions in decreasing order of organisational impact is: For several days Ram Lal, a night shift supervisor, had A. 4, 2, 3, 1 B. 4, 3, 2, 1 been trying to seek and appointment with C. 4, 3, 1, 2 D. 4, 1, 2, 3 Mr. Thakur about his wages. He was disgruntled, not E. 2, 3, 1, 4 only over his failure to see the president, but also over the lack of discussions about his wage contract prior to 73. The most likely premise behind Mr. Thakur’s its being effected. As a family man with six dependents, initiative regarding individualised meetings with he felt his weekly wage should be higher than that granted the supervisors seems to be to him. A. Employee related policies should allow scope for bargaining by employees. Last Thursday afternoon Ram Lal stopped by the B. Involvement of company’s president in wage president’s office and tried to see him. Mr.Thakur’s problems of employees will lead to a better secretary refused his request on the grounds that Mr. goodwill towards the management among the Thakur was busy. Infuriated, Ram Lal stormed into the workers. president’s office and confronted the startled Mr. Thakur, C. Individual agreements with supervisors would with stood up and told Ram Lal to get out of his office allow the management to prevent any possible and express his grievance through official channel. Ram collective action by the supervisors. Lal took a swing at the president, who in turn punched D. Management will be able to force supervisors Ram Lal on the jaw and knocked him unconscious. to accept lesser wages individually in this way. E. He would be able to know who the trouble makers in the plant are by interacting with the supervisors. 14 XAT – 2008
  • 15. 74. Out of the following, which one seems to be the 2. Finance department - for not taken into most likely cause of Ram Lal’s grievance? confidence regarding the financial A. His disappointment with the management’s consequences of the wage contracts. philosophy of having one to one interaction as 3. Marketing department - for not being consulted the supervisors were in a way being forced to on the likely impact of the wage contracts on accept the wage contracts. the image of the company. B. His being in the night shift had worked to his 4. Quality control - for not being able to give inputs disadvantage as he could not interact with the to Mr. Thakur on how to improve quality of steel management regarding his problem. making process. C. He was not allowed to meet chairman of the 5. Personnel department - for it was their work to board of directors of the company. oversee wage policies for employees and they D. Employment in the night shift forced him to stay had been ignored by Mr. Thakur. away from his family during the day time and A. 1 + 2 + 3 B. 1 + 4 + 5 therefore he could not interact with his family C. 1 + 2 + 5 D. 1 + 3 + 4 members much. E. 3 + 4 + 5 E. All of these. 76. Which of the following managerial attributes does 75. Apart from the supervisors working the night shift, Mr. Thakur seem to lack the most? executives of which department will have most A. Emotional instability under pressure. justified reasons to be disgruntled with Mr. B. Proactive problem solving. Thakur’s initiative? C. Ethical behaviour. 1. Production department - for not being consulted D. Emotional stability under pressure. regarding the behaviour of the supervisors on E. Independent decision making. the shop floor. SECTION C: DATA INTERPRETATION AND QUANTITATIVE ABILITY Note: All units of measurement are in centimetres, A. Returns of stock X are directly proportional to unless otherwise specified. mutual fund Y. 77. Four digits of the number 29138576 are omitted B. Average returns from stock X and mutual fund so that the result is as large as possible. Y are the same. The largest omitted digit is C. Stock X is less volatile than mutual fund Y. A. 9 B. 8 D. Stock X is more volatile than mutual fund Y. C. 7 D. 6 E. Stock X is inversely proportional to mutual fund E. 5 Y. 78. Interpret relationship between the returns of Stock Question 79 - 80: In second year, students at a business X and Mutual Fund Y based on the following graph, school can opt for Systems, Operations, or HR electives where percentage return of Stock X and Mutual only. The number of girls opting for Operations and the Fund Y are given for sixteen days of a month. number of boys opting for Systems electives is 37. 1.2 Twenty-two students opt for operations electives. Twenty 1 girls opt for Systems and Operations electives. Percentage Return The number of students opting for Systems electives 0.8 and the number of boys opting for Operations electives 0.6 is 37. Twenty-five students opt for HR electives. 0.4 79. The number of students in the second year is ___? 0.2 A. 73 B. 74 0 C. 76 D. 53 1 2 3 4 5 6 7 8 9 10 11 12 13 14 15 16 E. 54 Day of the Month Stock X Mutual Fund Y XAT – 2008 15
  • 16. 80. If 20% of the girls opt for HR electives, then the A. Using the given statement, only conclusion I total number of boys in the second year is ___? can be derived. A. 50 B. 52 B. Using the given statement, only conclusion C. 51 D. 53 II can be derived. C. Using the given statement, only conclusion E. 54 III can be derived. 81. ABCD is a rectangle with AD = 10. P is a point on D. Using the given statement, all conclusions BC such that ∠ APD = 90°. If DP = 8 then the I, II and III can be derived. length of BP is ___? E. Using the given statement, none of the three conclusions I, II and III can be derived. A. 6.4 B. 5.2 C. 4.8 D. 3.6 b 86. An operation “#” is defined by a # b = 1– E. None of the above Conclusion I. (2 # 1) # (4 # 3) = –1 a 82. Rajiv is a student in a business school. After every Conclusion II. (3 # 1) # (4 # 2) = –2 Conclusion III. (2 # 3) # (1 # 3) = 0 test he calculates his cumulative average. QT and OB were his last two tests.83 marks in QT 87. A, B, C and D are whole numbers such that increased his average by 2. 75 marks in OB further A + B +C = 118 B + C + D = 156 increased his average by 1. Reasoning is the next C + D + A = 166 D + A + B = 178 test, if he gets 51 in Reasoning, his average will Conclusion I. be ___? A is the smallest number and A = 21. A. 59 B. 60 Conclusion II. C. 61 D. 62 D is the smallest number and D = 88. Conclusion III. E. 63 B is the smallest number and B = 56. 83. ABCD is a quadrilateral. The diagonals of ABCD 88. Let X = {a, b, c} and Y = {1, m}. intersect at the point P. The area of the triangles Consider the following four subsets of X × Y. APD and BPC are 27 and 12, respectively. If the F1 = {(a, 1), (a, m), (b, 1), (c, m)} areas of the triangles APB and CPD are equal F2 = {(a, 1), (b, 1), (c, 1)} then the area of triangle APB is F3 = {(a, 1), (b, m), (c, m)} A. 12 B. 15 F4 = {(a, 1), (b, m)} C. 16 D. 21 Which one, amongst the choices given below, is a representation of functions from X to Y? E. 18 A. F2 and F3 B. F1, F2 and F3 84. If F (x, n) be the number of ways of distributing C. F2, F3 and F4 D. F3 and F4 “x” toys to “n” children so that each child receives E. None of the above at the most 2 toys, then F(4, 3) = ___? 89. In the figure, number in any cell is obtained by A. 2 B. 6 adding two numbers in the cells directly below it. C. 3 D. 4 For example, 9 in the second row is obtained by E. 5 adding the two number 4 and 5 directly below it. The value of X - Y is 85. In a cricket match, Team A scored 232 runs without losing a wicket. The score consisted of byes, wides 68 and runs scored by two opening batsmen: Ram and Shyam. The runs scored by the two batsmen Y + 29 are 26 times wides. There are 8 more byes than wides. If the ratio of the runs scored by Ram and Shyam is 6 : 7, then the runs scored by Ram is A. 88 B. 96 9 C. 102 D. 112 E. None of the above. Y 4 5 2 X For question 86 and 87, a statement is followed by A. 2 B. 4 three conclusions. Select the answer from the C. 3 D. 5 following options. E. 6 16 XAT – 2008
  • 17. 90. For each p > 1, a sequence {An} is defined by 97. If [x] denotes the greatest integer < = x, then A0 = 1 and An = pn + (–1)n An – 1 for each n ≥1. For  1  1 1   1 2   1 98  how many integer values of p, 1000 is a term of  3  +  3 + 99  +  3 + 99  + ...  3 + 99  =         the sequence? A. 98 B. 33 A. 8 B. 7 C. 67 D. 66 C. 5 D. 4 E. 34 E. None of the above. 98. ABCD is a square. P is the midpoint of AB. 91. If o < p< 1 then roots of the equation The line passing through A and Perpendicular to (1 – p) x2 + 4x + p = 0 are ___? DP intersects the diagonal at Q and BC at R. A. Both 0 If AB = 2 then PR = ___? B. Real and both negative 1 A. B. 2 C. Imaginary 2 D. Real and both positive 3 C. 1 D. E. Real and of opposite sign. 2 E. None of the above. 92. If x > 0, the minimum value of 99. Two circles of radius 1 cm. touch at point P. A 6 third circle is drawn through the points A, B, and  1  6 1  x + x  – x + 6 –2 C such that PA is the diameter of the first circle,    x  and BC - perpendicular to AP - is the diameter of is ___? 3  1  3 1  the second circle. The radius of the third circle is x + x  + x + 3  ___ cms.    x  9 7 A. 1 B. 2 A. B. C. 3 D. 6 5 4 E. None of the above. 5 10 C. D. 93. The number of possible real solution(s) of y in 3 2 equation y2 – 2y cos x + 1 = 0 is____? E. 2 A. 0 B. 1 100. Consider a sequence –6, –12, 18, 24, –30, –36, C. 2 D. 3 42,...... If sum of the first n terms of the sequence E. None of the above. is 132, then the value of n is ___? 94. In a triangle ABC, AB = 3, BC = 4 and CA = 5. A. 11 B. 22 Point D is the midpoint of AB, point E is on the C. 13 D. 18 segment AC and point F is on the segment BC. E. 24 If AE = 1.5 and BF = 0.5 then ∠ DEF = Question No. 101 - 102 are followed by two A. 30° B. 60° statements labelled as I and II. You have to decide if C. 45° D. 75° these statements are sufficient to conclusively E. Cannot be determined answer the question. Choose the appropriate answer  1  from options given below: 95. If 3f ( x + 2 ) + 4f   = 4x,x ≠ –2 , then f(4) =  x +2 A .If Statement I alone is sufficient to answer the A. 7 B. 52 / 7 question. C. 8 D. 56 / 7 B. If Statement II alone is sufficient to answer the E. None of the above. question. 96. A train left station X at A hour B minutes. It reached C. If Statement I and Statement II together are station Y at B hour C minutes on the same day, sufficient but neither of the two alone is after travelling for C hour A minutes (clock shows sufficient to answer the question. time from 0 hours to 24 hours). Number of possible D. If either Statement I or Statement II alone is value(s) of A is ___. sufficient to answer the question. A. 3 B. 2 E. Both Statement I and Statement II and C. 1 D. 0 E. None of the above. insufficient to answer the question. XAT – 2008 17